Top Banner
INSIGHTSIAS SIMPLYFYING IAS EXAM PREPARATION INSTA Tests DAYS 13 to 16 INSTA Revision Plan 3.0 - 2020 Copyright © by Insights IAS All rights are reserved. No part of this document may be reproduced, stored in a retrieval system or transmitted in any form or by any means, electronic, mechanical, photocopying, recording or otherwise, without prior permission of Insights IAS. OFFLINE Centres at BENGALURU | DELHI | HYDERABAD For more visit: www.INSIGHTSONINDIA.com SOLUTIONS
83

INSTA Revision Plan 3.0 - 2020 INSTA Tests - INSIGHTSIAS...2020/08/04  · palm oil. Palm oil contributes 70% of vegetable oil import and is one of the cheapest oil due to high productivity

Oct 09, 2020

Download

Documents

dariahiddleston
Welcome message from author
This document is posted to help you gain knowledge. Please leave a comment to let me know what you think about it! Share it to your friends and learn new things together.
Transcript
Page 1: INSTA Revision Plan 3.0 - 2020 INSTA Tests - INSIGHTSIAS...2020/08/04  · palm oil. Palm oil contributes 70% of vegetable oil import and is one of the cheapest oil due to high productivity

INSIGHTSIAS SIMPLYFYING IAS EXAM PREPARATION

INSTA Tests DAYS 13 to 16

INSTA Revision Plan 3.0 - 2020

Copyright © by Insights IAS All rights are reserved. No part of this document may be reproduced, stored in a retrieval system or transmitted in any form or by any means, electronic, mechanical, photocopying, recording or otherwise, without prior permission of Insights IAS.

OFFLINE Centres at BENGALURU | DELHI | HYDERABAD

For more visit: www.INSIGHTSONINDIA.com

SOLUTIONS

Page 2: INSTA Revision Plan 3.0 - 2020 INSTA Tests - INSIGHTSIAS...2020/08/04  · palm oil. Palm oil contributes 70% of vegetable oil import and is one of the cheapest oil due to high productivity
Page 3: INSTA Revision Plan 3.0 - 2020 INSTA Tests - INSIGHTSIAS...2020/08/04  · palm oil. Palm oil contributes 70% of vegetable oil import and is one of the cheapest oil due to high productivity

www.insightsonindia.com 1 INSTA Revision 3.0

INSIGHTSIAS SIMPLYFYING IAS EXAM PREPARATION

DAY – 13

1. Arrange the following hills from East to West

1. Rajmahal hills

2. Ramgarh hills

3. Ajanta hills

4. Satmala range

Select the correct answer using the code given below:

(a) 2 4 1 3

(b) 2 3 1 4

(c) 1 2 4 3

(d) 1 2 3 4

Solution: C

2. Which of the following passes connects with China?

1. Shipki La

2. Lipu Lekh

3. Nathu la Pass

Select the correct answer using the code given below:

(a) 1 and 2 only

Page 4: INSTA Revision Plan 3.0 - 2020 INSTA Tests - INSIGHTSIAS...2020/08/04  · palm oil. Palm oil contributes 70% of vegetable oil import and is one of the cheapest oil due to high productivity

www.insightsonindia.com 2 INSTA Revision 3.0

INSIGHTSIAS SIMPLYFYING IAS EXAM PREPARATION

(b) 1 and 3 only

(c) 2 and 3 only

(d) 1, 2 and 3

Solution: D

• Nathu La Pass It is located in the state of Sikkim. This famous pass is located

in the India- China border was reopened in 2006. It forms a part of an offshoot

of the ancient silk route. It is one of the trading border posts between India and

China.

• Shipki La Pass It is located through Sutlej Gorge. It connects Himachal

Pradesh with Tibet. It is India’s third border post for trade with China after Lipu

Lekh and Nathula Pass.

• Jelep La Pass This pass passes through the Chumbi valley. It connects

Sikkim with Lhasa, the capital of Tibet.

• Chang-La It is a high mountain pass in the Greater Himalayas. It connects

Ladakh with Tibet.

• Debsa Pass: Spiti Valley and Parvati Valley It joins Spiti Valley and Parvati

Valley. It is a high mountain pass in between the Kullu and Spiti of Himachal

Pradesh. It is a bypass route of Pin-Parvati Pass.

• Rohtang Pass: Kullu-Lahul-Spiti This is located in the state of Himachal

Pradesh. It has excellent road transportation. This pass connects Kullu, Spiti,

and Lahul.

• Bomdi-La: Arunachal Pradesh-Lhasa The Bomdi-La pass connects

Arunachal Pradesh with Lhasa, the capital city of Tibet. It is located in the east

of Bhutan.

• Zoji La: Srinagar- Kargil & Leh It connects Srinagar with Kargil and Leh.

Beacon Force of Border Road Organization is responsible for clearing and

maintaining the road, especially during the winter.

• Lipu Lekh: Uttarakhand-Tibet It is located in Uttarakhand. It connects

Uttarakhand with Tibet. This pass is an important border post for trade with

China. The pilgrims for Manasarovar travel through this pass.

3. Consider the following statements

1. A substantial portion of India’s requirement of edible oil is met through

import of palm oil.

2. India’s vegetable oil economy is world’s fourth largest after USA, China &

Brazil.

3. Palm oil contributes 30% of vegetable oil import of India

Page 5: INSTA Revision Plan 3.0 - 2020 INSTA Tests - INSIGHTSIAS...2020/08/04  · palm oil. Palm oil contributes 70% of vegetable oil import and is one of the cheapest oil due to high productivity

www.insightsonindia.com 3 INSTA Revision 3.0

INSIGHTSIAS SIMPLYFYING IAS EXAM PREPARATION

Which of the statements given above is/are correct?

(a) 1 and 3 only

(b) 3 only

(c) 2 and 3 only

(d) 1 and 2 only

Solution: D

India is one of the major oilseeds grower and importer of edible oils. India’s

vegetable oil economy is world’s fourth largest after USA, China & Brazil.

A substantial portion of India’s requirement of edible oil is met through import of

palm oil. Palm oil contributes 70% of vegetable oil import and is one of the cheapest

oil due to high productivity per hectare.

India is the world’s largest importer of palm oil, driving 23 per cent of total global

demand from plantations in Indonesia and Malaysia.

https://www.thehindu.com/business/avoid-oil-imports-from-malaysia-trade-

body/article29760672.ece

4. Consider the following statements

1. The Thal Ghat is located on Mumbai–Nashik route.

2. Pal Ghat joins the Madurai city in Tamil Nadu with Kottayam district in

Kerala.

Which of the statements given above is/are correct?

(a) 1 only

(b) 2 only

(c) Both 1 and 2

(d) Neither 1 nor 2

Solution: A

Shencottah Gap: Madurai-Kottayam It is located in the Western Ghats. It joins

the Madurai city in Tamil Nadu with Kottayam district in Kerala.

• The second largest gap in Western Ghats which is situated five kilometres from

town is known by its name that is Shencottah Gap road rail lines pass through

this gap which connect Shencottah with Punalur.

Page 6: INSTA Revision Plan 3.0 - 2020 INSTA Tests - INSIGHTSIAS...2020/08/04  · palm oil. Palm oil contributes 70% of vegetable oil import and is one of the cheapest oil due to high productivity

www.insightsonindia.com 4 INSTA Revision 3.0

INSIGHTSIAS SIMPLYFYING IAS EXAM PREPARATION

Bhor Ghat Bhor Ghat or Bor Ghat or Bhore Ghaut is a mountain passage located

between Palasdari and Khandala for railway and between Khopoli and Khandala

on the road route in Maharashtra, India situated on the crest of the western Ghats.

It is located at an elevation of four hundred and forty-one meters’ elevation above sea

level.

• The ghat has a bit of historical evidence. The ghat was the ancient route

developed by Satavahana to connect the ports of Choul, Revdanda Panvel etc.

on the Konkan coast and the surrounding areas on the Deccan plateau. Today

the ghat plays a massive part of the Great Indian Peninsula Railway laid from

Mumbai to Pune.

Thal Ghat Thal Ghat (also called Thul Ghat or Kasara Ghat) is a ghat section

(mountain incline or slope) in the Western Ghats near the town of Kasara in

Maharashtra. The Thal Ghat is located on the busy Mumbai–Nashik route, and is

one of the four major routes, rail and road routes, and leading into Mumbai. The

railway line, which passes through the ghat is the steepest in India with a gradient

of 1 in 37

Pal Ghat

• The Palakkad Gap is located in the Western Ghats between the states of Tamil

Nadu and Kerala. India at an elevation of about 140 m. The mountain pass is

located between Nilgiri Hills in the north and Anaimalai Hills towards the south

and connects Coimbatore in Tamil Nadu with Palakkad in Kerala. The

mountain pass was an important instrument for human migration across India’s

southern tip throughout settled history.

5. Which of the following parameters is/are considered to rank institutions across

India under National Institute ranking framework (NIRF)?

1. Teaching learning and resources

2. Research, consulting and collaborative performance

3. Outreach and inclusivity

Select the correct answer using the code given below

(a) 2 only

(b) 1 and 3 only

(c) 3 only

(d) 1, 2 and 3

Solution: D

National Institute Ranking Framework (NIRF) is a methodology to rank

institutions across the country.

Page 7: INSTA Revision Plan 3.0 - 2020 INSTA Tests - INSIGHTSIAS...2020/08/04  · palm oil. Palm oil contributes 70% of vegetable oil import and is one of the cheapest oil due to high productivity

www.insightsonindia.com 5 INSTA Revision 3.0

INSIGHTSIAS SIMPLYFYING IAS EXAM PREPARATION

It provides credible official rankings of India’s higher educational institution on a

set of parameters. The National Institutional Ranking Framework (NIRF) was

approved by the MHRD and launched by the Minister of Human Resource

Development on September 29, 2015.

The parameters are under the five broad headings:

1. Teaching learning and resources

2. Research, consulting and collaborative performance

3. Graduation outcomes

4. Outreach and inclusivity

5. Perception

6. Which of the following is/are artificial lakes?

1. Kodaikanal Lake

2. Bhojtal Lake

3. Kankaria Lake

4. Hamirsar Lake

Select the correct answer using the code given below:

(a) 1 and 2 only

(b) 3 and 4 only

(c) 1, 2 and 4 only

(d) 1, 2, 3 and 4

Solution: D

Hamirsar Lake

• Type: Artificial Lake

• Surface area: 11 Hectare

• State: Gujarat

Kankaria Lake

• Surface area: Artificial Lake

• State: Gujarat

Page 8: INSTA Revision Plan 3.0 - 2020 INSTA Tests - INSIGHTSIAS...2020/08/04  · palm oil. Palm oil contributes 70% of vegetable oil import and is one of the cheapest oil due to high productivity

www.insightsonindia.com 6 INSTA Revision 3.0

INSIGHTSIAS SIMPLYFYING IAS EXAM PREPARATION

Artificial Lakes are constructed for industrial or agricultural use, for hydro-electric

power generation or domestic water supply, or for aesthetic or recreational purposes.

• Guru Gobind Sagar is an example of an artificial lake. It supports the Bhakra

Nangal Hydel Project.

• Kodaikanal Lake in Tamil Nadu is another example of Artificial (manmade)

Lake.

• Bhojtal Lake- Madhya Pradesh

› Also known as Upper Lake lies on the western side of the capital city of Madhya

Pradesh, Bhopal.

› The largest artificial lake in Asia.

7. Consider the following statements:

1. Lake Superior is the world’s largest freshwater lake in terms of volume.

2. Lake Baikal is the world’s deepest lake.

3. The Dead Sea is the saltiest body of water on Earth.

Which of the statements given above is/are correct?

(a) 2 only

(b) 1 and 3 only

(c) 2 and 3 only

(d) 1, 2 and 3

Solution: A

Lake Baikal is the world’s largest freshwater lake in terms of volume.

Lake Baikal is the largest freshwater lake in the world (by volume) and the

world’s deepest lake.

The salty Caspian Sea has the greatest surface area of any lake at 143,200 square

miles (370,886 square kilometers).

The Dead Sea, nestled on the borders of Jordan and Israel. The water here is around

10 times saltier than sea water. However, it is only the fifth saltiest body of water on

Earth.

8. Consider the following statements regarding Central Electricity Regulatory

Commission (CERC)

1. It is an autonomous body under Ministry of Power

Page 9: INSTA Revision Plan 3.0 - 2020 INSTA Tests - INSIGHTSIAS...2020/08/04  · palm oil. Palm oil contributes 70% of vegetable oil import and is one of the cheapest oil due to high productivity

www.insightsonindia.com 7 INSTA Revision 3.0

INSIGHTSIAS SIMPLYFYING IAS EXAM PREPARATION

2. It was instituted primarily to regulate the tariff of Power Generating

companies.

Which of the statements given above is/are correct?

(a) 1 only

(b) 2 only

(c) Both 1 and 2

(d) Neither 1 nor 2

Solution: B

• CERC is a statutory body functioning under section 76 of the Electricity Act

2003. It functions with a quasi-judicial status under the Ministry of Power.

• CERC was initially constituted under the Electricity Regulatory Commissions

Act, 1998 on 24th July, 1998.

Functions

• CERC was instituted primarily to regulate the tariff of Power Generating

companies (government owned as well as other companies with interstate

transmission.)

It works for

o Rationalization of electricity tariffs

o Transparent policies regarding subsidies

o Promotion of efficient and environmentally benign policies

o Electricity Tariff regulation

9. Arrange the following cities from west to east

1. Nagpur

2. Ujjain

3. Udaipur

4. Junagadh

Select the correct answer using the code given below:

(a) 4 1 3 2

(b) 4 3 2 1

(c) 1 4 2 3

(d) 1 3 2 4

Page 10: INSTA Revision Plan 3.0 - 2020 INSTA Tests - INSIGHTSIAS...2020/08/04  · palm oil. Palm oil contributes 70% of vegetable oil import and is one of the cheapest oil due to high productivity

www.insightsonindia.com 8 INSTA Revision 3.0

INSIGHTSIAS SIMPLYFYING IAS EXAM PREPARATION

Solution: B

10. Consider the following statements regarding Traditional Knowledge Digital

Library (TKDL)

1. It is a community based informal traditional knowledge repository

2. It is supported by Ministry of AYUSH

3. TKDL acts as a bridge between the traditional knowledge information

existing in local languages and the patent examiners at IPOs.

Which of the statements given above is/are correct?

(a) 1 and 2 only

(b) 3 only

(c) 1 and 3 only

(d) 2 and 3 only

Solution: D

Traditional Knowledge Digital Library (TKDL) is an Indian digital knowledge

repository of the traditional knowledge, especially about medicinal plants and

formulations used in Indian systems of medicine.

• It is a pioneer initiative of India to prevent misappropriation of country’s

traditional medicinal knowledge at International Patent Offices. It is a

collaborative project of the Council of Scientific and Industrial Research

(CSIR) and Ministry of AYUSH.

Page 11: INSTA Revision Plan 3.0 - 2020 INSTA Tests - INSIGHTSIAS...2020/08/04  · palm oil. Palm oil contributes 70% of vegetable oil import and is one of the cheapest oil due to high productivity

www.insightsonindia.com 9 INSTA Revision 3.0

INSIGHTSIAS SIMPLYFYING IAS EXAM PREPARATION

• TKDL acts as a bridge between the traditional knowledge information existing in

local languages and the patent examiners at IPOs.

• The focus of TKDL was on breaking the language and format barriers by

scientifically converting and structuring the available TK in International Patent

Classification.

• The TKDL contains documentation of publicly available traditional knowledge

(TK) that relates to Ayurveda, Unani, Siddha and Yoga and is in digitized

format.

11. Arrange the following cities from North to South

1. Shimla

2. Dehradun

3. Agra

4. New Delhi

Select the correct answer using the code given below:

(a) 2 4 1 3

(b) 2 3 1 4

(c) 1 2 4 3

(d) 1 2 3 4

Solution: C

Page 12: INSTA Revision Plan 3.0 - 2020 INSTA Tests - INSIGHTSIAS...2020/08/04  · palm oil. Palm oil contributes 70% of vegetable oil import and is one of the cheapest oil due to high productivity

www.insightsonindia.com 10 INSTA Revision 3.0

INSIGHTSIAS SIMPLYFYING IAS EXAM PREPARATION

12. Consider the following islands of Andaman and Nicobar islands:

1. Barren Island

2. Little Andaman

3. Car Nicobar Island

Arrange the above given islands in the direction from South to North:

(a) 3-2-1

(b) 3-1-2

(c) 1-2-3

(d) 1-3-2

Solution: A

Page 13: INSTA Revision Plan 3.0 - 2020 INSTA Tests - INSIGHTSIAS...2020/08/04  · palm oil. Palm oil contributes 70% of vegetable oil import and is one of the cheapest oil due to high productivity

www.insightsonindia.com 11 INSTA Revision 3.0

INSIGHTSIAS SIMPLYFYING IAS EXAM PREPARATION

13. Consider the following statements regarding Safe City Project

1. It is a comprehensive plan with the purpose of strengthening safety and

security of women in public places

2. It will be implemented in few selected cities under Nirbhaya Fund.

Which of the statements given above is/are correct?

(a) 1 only

(b) 2 only

(c) Both 1 and 2

(d) Neither 1 nor 2

Solution: C

Safe City Project is a comprehensive plan with the purpose of strengthening safety

and security of women in public places.

It will be implemented in 8 selected cities under Nirbhaya Fund. The eight cities

include Mumbai, Delhi, Kolkata, Chennai, Bengaluru, Hyderabad, Ahmedabad

and Lucknow.

The project would be implemented as a Centrally Sponsored Scheme with Centre and

State sharing the funding in 60:40 ratio.

14. Consider the following statements regarding the Andhra Pradesh state:

1. The state has the longest coastline among all the states of India.

2. Rice is a major food crop in Andhra Pradesh, contributing about 77 percent

of the food grain production.

Which of the statements given above is/are correct?

(a) 1 only

(b) 2 only

(c) Both 1 and 2

(d) Neither 1 nor 2

Solution: B

Andhra Pradesh

• The state has the second longest coastline of 974 km (605 mi) among all the

states of India, second only to Gujarat.

Page 14: INSTA Revision Plan 3.0 - 2020 INSTA Tests - INSIGHTSIAS...2020/08/04  · palm oil. Palm oil contributes 70% of vegetable oil import and is one of the cheapest oil due to high productivity

www.insightsonindia.com 12 INSTA Revision 3.0

INSIGHTSIAS SIMPLYFYING IAS EXAM PREPARATION

• Agriculture is the main occupation of about 62 per cent of the people in the

state. Rice is a major food crop and staple food contributing about 77 percent of

the food grain production.

• Andhra Pradesh is situated on the country’s southeastern coast. It is the eighth

largest state in the country covering an area of 160,205 sq. km. According to

2011 census, the state is tenth largest by population, with 49,386,799

inhabitants.

15. Global Economic Prospects is released by

(a) United Nations

(b) International Monetary Fund

(c) World Economic Forum

(d) None of the above

Solution: D

The Global Economic Prospects (GEP) is the World Bank’s semi-annual flagship

publication on the state of the world economy. In a nutshell, the outlook for the

global economy has darkened, amid slowing activity and heightened downside risks.

16. Consider the following statements regarding the Himachal Pradesh State:

1. It is called the “Fruit Bowl” of the country.

2. The territory of this state has parts of the Shivaliks, Inner Himalayas and

the Greater Himalayas.

3. Tourism contributes about 25 per cent to state GDP.

Which of the statements given above is/are correct?

(a) 1 and 2 only

(b) 1 and 3 only

(c) 2 and 3 only

(d) 1, 2 and 3

Solution: A

Himachal Pradesh

• Himachal Pradesh, a beautiful land is situated in the lap of the western

Himalayas, located between 30°.22’ and 33°. 12’ North latitude and between

Page 15: INSTA Revision Plan 3.0 - 2020 INSTA Tests - INSIGHTSIAS...2020/08/04  · palm oil. Palm oil contributes 70% of vegetable oil import and is one of the cheapest oil due to high productivity

www.insightsonindia.com 13 INSTA Revision 3.0

INSIGHTSIAS SIMPLYFYING IAS EXAM PREPARATION

75°.47’ and 79°. 41 East longitude. This wonderful land inhabited by people of

various castes and religion groups, is also called as “Dev Bhumi: the abode of

gods and goddesses”. Himachal drives its name from the Himalayas, literally

means “land of snowy mountains”. The territory of this state can be divided

into three zones, Outer Himalayas or the Shivaliks, Inner Himalayas or Mid-

Mountains and the Greater Himalayas or Alpine Zone.

Agriculture

• Agriculture provides direct employment to about 70 per cent of the main working

population of the state. Income from agriculture and allied sectors accounts for

nearly 22.1 per cent of the total state domestic product. Out of the total

geographical area of 55, 673 sq. km., area of operational holding is about 9.79

lakh hectares.

Horticulture

• Himachal Pradesh is called the “Fruit Bowl” of the country. Horticulture plays

an important role in the agrarian economy.

Forestry

• Forest in Himachal Pradesh covers an area of 37, 033 sq. km which comes to

66.52 per cent of the total geographical area of the state. The strategy of

government in forestry management is conservation along with rational

utilization and side by side expanding its base. Forest plantation is being carried

out under productive forestry scheme and soil conservation schemes.

Tourism contributes 6.8 per cent to state GDP and it is set to increase.

17. Consider the following statements regarding the Chumbi valley:

1. It is located in the state of Uttarakhand.

2. It is formed by the passage of the River Bhagirathi.

Which of the statements given above is/are correct?

(a) 1 only

(b) 2 only

(c) Both 1 and 2

(d) Neither 1 nor 2

Solution: D

Chumbi Valley, valley in the eastern Great Himalaya Range of the southern Tibet

Autonomous Region, China. It is situated on a small south-pointing protuberance

of territory between Bhutan (east) and Sikkim state, India (west). Formed by the

passage of the Amo (Torsa) River, which rises below Tang Pass and flows south into

Page 16: INSTA Revision Plan 3.0 - 2020 INSTA Tests - INSIGHTSIAS...2020/08/04  · palm oil. Palm oil contributes 70% of vegetable oil import and is one of the cheapest oil due to high productivity

www.insightsonindia.com 14 INSTA Revision 3.0

INSIGHTSIAS SIMPLYFYING IAS EXAM PREPARATION

Bhutan, the valley has an average elevation of 9,500 feet (2,900 metres), forested

slopes, and a pleasant climate most of the year.

It is connected to Sikkim to the southwest through the mountain passes of Nathu

La and Jelep La. It is a disputed territory between China and Bhutan.

Important for prelims because of Doklam standoff.

18. Consider the following statements regarding Bank Boards Bureau (BBB)

1. It appoints Board of Directors in PSBs and Financial Institutions.

2. It builds a data bank containing data relating to the performance of

PSBs/FIs, its senior management.

Which of the statements given above is/are correct?

(a) 1 only

(b) 2 only

(c) Both 1 and 2

(d) Neither 1 nor 2

Solution: C

• Banks Board Bureau is an autonomous body of the Government of India tasked

to improve the governance of Public Sector Banks.

• It was set up under the government’s Indradhanush programme. It was

conceived by the PJ Nayak committee.

Composition

• BBB has representatives from government and RBI apart from independent

banking professionals. It is housed in RBI’s Central Office in Mumbai.

Functions of the Banks Board Bureau:

• Selection and appointment of Board of Directors in PSBs and Financial

Institutions (Whole-time Directors and Non-Executive Chairman);

• Advising the central government in matters of banking governance at the higher

levels and their training and management.

• Building a data bank containing data relating to the performance of PSBs/FIs,

its senior management and the Board of Directors and share the same with

Government;

19. Rutland Island, recently seen in news, is located in

(a) Andaman and Nicobar

Page 17: INSTA Revision Plan 3.0 - 2020 INSTA Tests - INSIGHTSIAS...2020/08/04  · palm oil. Palm oil contributes 70% of vegetable oil import and is one of the cheapest oil due to high productivity

www.insightsonindia.com 15 INSTA Revision 3.0

INSIGHTSIAS SIMPLYFYING IAS EXAM PREPARATION

(b) Lakshadweep

(c) Gulf of Kutch

(d) Ganga Brahmaputra Delta

Solution: A

Rutland Island is an island of the Andaman Islands. It belongs to the South

Andaman administrative district.

The National Board of Wildlife (NBWL) has approved Rutland Island in South

Andaman to be the site for the country’s long-range missile test facility.

20. With reference to the Renewable Energy Certificates (REC), consider the

following statements

1. It is a mechanism is a market based instrument to promote renewable

sources of energy and development of market in electricity.

2. RECs are not eligible to trade in India

3. RECs represent 1mw-hour of power produced from a renewable energy

source

Which of the statements given above is/are correct?

(a) 2 only

(b) 1 and 3 only

(c) 2 and 3 only

(d) 1 only

Solution: B

The Renewable Energy Certificates (REC) mechanism is a market based

instrument to promote renewable sources of energy and development of market in

electricity.

Utilities that are unable to fulfil their RPO can meet their obligations by buying

certificates known as Renewable Energy Certificates (REC). RECs represent 1mw-

hour of power produced from a renewable energy source and are tradable at power

exchanges.

It is classified into Solar REC and non-solar REC.REC market was launched in

2010.

National Load Despatch Centre (NLDC) has been designated as the Central Agency

for coordination and implementation of Renewable Energy Certificate Mechanism.

Page 18: INSTA Revision Plan 3.0 - 2020 INSTA Tests - INSIGHTSIAS...2020/08/04  · palm oil. Palm oil contributes 70% of vegetable oil import and is one of the cheapest oil due to high productivity

www.insightsonindia.com 16 INSTA Revision 3.0

INSIGHTSIAS SIMPLYFYING IAS EXAM PREPARATION

In India, RECs are traded on the Indian Energy Exchange (IEX) and the Power

Exchange of India Ltd (PXIL).

21. Consider the following pairs of places in news and their states:

Place State 1. Belum caves : Madhya Pradesh 2. Lothal : Gujarat 3. Bogibeel Bridge : Arunachal Pradesh

Which of the pairs given above is/are correctly matched?

(a) 1 and 3 only

(b) 2 only

(c) 2 and 3 only

(d) 1 and 2 only

Solution: B

The Belum Caves is the largest and longest cave system open to the public on the

Indian subcontinent, known for its speleothems, such as stalactite and stalagmite

formations. The Belum Caves have long passages, galleries, spacious caverns with

fresh water and siphons. It is located in the state of Andhra Pradesh.

https://www.thehindu.com/news/national/andhra-pradesh/belum-caves-festival-

postponed/article30413624.ece

Lothal was one of the southernmost cities of the ancient Indus Valley Civilization,

located in the Bhal region of the modern state of Gujarat.

https://www.thehindubusinessline.com/news/science/indias-first-maritime-

museum-coming-up-at-lothal-in-gujarat/article30139189.ece

Bogibeel bridge is a combined road and rail bridge over the Brahmaputra River in

the northeastern Indian state of Assam

https://www.livemint.com/Politics/zMhowFzkONQkLllh9ycZtK/PM-Modi-

inaugurates-Bogibeel-Indias-longest-railroad-brid.html

22. Koundinya Wildlife Sanctuary, sometimes seen in news, is located in:

(a) Karnataka

(b) Kerala

(c) Tamil Nadu

Page 19: INSTA Revision Plan 3.0 - 2020 INSTA Tests - INSIGHTSIAS...2020/08/04  · palm oil. Palm oil contributes 70% of vegetable oil import and is one of the cheapest oil due to high productivity

www.insightsonindia.com 17 INSTA Revision 3.0

INSIGHTSIAS SIMPLYFYING IAS EXAM PREPARATION

(d) Andhra Pradesh

Solution: D

Kaundinya Wildlife Sanctuary is a wildlife sanctuary and an elephant reserve

situated in Andhra Pradesh, India. It is the only sanctuary in Andhra Pradesh with

a population of Asian elephants, which migrated after 200 years from neighbouring

regions.

https://www.thehindu.com/news/national/andhra-pradesh/4-poachers-nabbed-

in-koundinya-wildlife-sanctuary/article32096013.ece

23. Consider the following statements regarding Tiger Census, 2018

1. Tiger census conducted for every four years

2. Both cubs and adult tigers are counted in the census

3. It is conducted under the overall guidance of Global Tiger Forum

Which of the statements given above is/are correct?

(a) 1 only

(b) 1 and 2 only

(c) 1, 2 and 3

(d) 1 and 3 only

Solution: A

Tiger Census 2018

• It is a four-yearly report, which gives out the number of big cats living in the

country.

• This is the fourth cycle of the tiger census. The first was conducted in 2006,

second in 2010 and third in 2014.

• In the census, only adult tigers are counted and do not include cubs. This is the

first time neighboring Nepal, Bangladesh and Bhutan are being roped in for the

census because they constitute a larger tiger range in the Indian subcontinent.

• It is conducted under the overall guidance of NTCA and Ministry of

Environment, Forest and Climate Change.

Page 20: INSTA Revision Plan 3.0 - 2020 INSTA Tests - INSIGHTSIAS...2020/08/04  · palm oil. Palm oil contributes 70% of vegetable oil import and is one of the cheapest oil due to high productivity

www.insightsonindia.com 18 INSTA Revision 3.0

INSIGHTSIAS SIMPLYFYING IAS EXAM PREPARATION

24. Consider the following statements regarding the Andaman and Nicobar Islands:

1. Little Andaman is separated from the Nicobar Islands by the Ten Degree

Channel.

2. Saddle Peak is located on Car Nicobar Island.

3. Narcondam volcanic island is located south of Barren Island.

Which of the statements given above is/are correct?

(a) 1 only

(b) 2 only

(c) 1 and 3 only

(d) 1, 2 and 3

Solution: A

Page 21: INSTA Revision Plan 3.0 - 2020 INSTA Tests - INSIGHTSIAS...2020/08/04  · palm oil. Palm oil contributes 70% of vegetable oil import and is one of the cheapest oil due to high productivity

www.insightsonindia.com 19 INSTA Revision 3.0

INSIGHTSIAS SIMPLYFYING IAS EXAM PREPARATION

Ten degrees Channel which lies between Little Andaman Island and the Car

Nicobar. It is so named as it lies on the 10-degree line of latitude, north of the

equator.

Saddle Peak or Saddle Hill is located on North Andaman Island in India’s Andaman

and Nicobar Islands. At 731 m, it is the highest point of the archipelago in the Bay

of Bengal. It is surrounded by Saddle Peak National Park

25. Dhimsa, a tribal dance, is largely practiced in which of the following region of

India?

(a) Northern India

(b) Eastern India

(c) Western India

(d) North-Eastern India

Solution: B

Dhimsa is a tribal dance performed by the Porja’ caste women. The dance is

originated in Odisha but has almost become a dance of Visakhapatnam, Andhra

Pradesh.

It is performed by a group of 15–20 women in a circle – praising the deity for the

welfare of their domestic life or at weddings for a happy married life.

DAY – 14

26. Consider the following statements regarding Effects of El Nino

1. Fish catches off the coast of South America were lower than in the normal

year.

2. Heavy rains in California, Ecuador, and the Gulf of Mexico.

3. Severe droughts occur in Australia, Indonesia, India and southern Africa.

Which of the statements given above is/are correct?

(a) 1 and 2 only

(b) 1 and 3 only

(c) 2 and 3 only

(d) 1, 2 and 3

Solution: D

Page 22: INSTA Revision Plan 3.0 - 2020 INSTA Tests - INSIGHTSIAS...2020/08/04  · palm oil. Palm oil contributes 70% of vegetable oil import and is one of the cheapest oil due to high productivity

www.insightsonindia.com 20 INSTA Revision 3.0

INSIGHTSIAS SIMPLYFYING IAS EXAM PREPARATION

Effects of El Nino

• The warmer waters had a devastating effect on marine life existing off the coast

of Peru and Ecuador.

• Fish catches off the coast of South America were lower than in the normal year

(Because there is no upwelling).

• Severe droughts occur in Australia, Indonesia, India and southern Africa.

• Heavy rains in California, Ecuador, and the Gulf of Mexico.

How El Nino impacts monsoon rainfall in India

• El Nino and Indian monsoon are inversely related.

• The most prominent droughts in India – six of them – since 1871 have been El

Nino droughts, including the recent ones in 2002 and 2009

• However, not all El Nino years led to a drought in India. For instance, 1997/98

was a strong El Nino year but there was no drought (Because of IOD).

• On the other hand, a moderate El Nino in 2002 resulted in one of the worst

droughts.

• El Nino directly impacts India’s agrarian economy as it tends to lower the

production of summer crops such as rice, sugarcane, cotton and oilseeds.

• The ultimate impact is seen in the form of high inflation and low gross domestic

product growth as agriculture contributes around 14 per cent to the Indian

economy.

27. Consider the following statements regarding Indian Ocean Dipole

1. The Indian Ocean Dipole (IOD) is defined by the difference in sea surface

temperature between two areas – a western pole in the Arabian Sea and an

eastern pole in the eastern Indian Ocean.

2. Negative IOD results in stronger than usual cyclonogenesis in Arabian sea.

Which of the statements given above is/are correct?

(a) 1 only

(b) 2 only

(c) Both 1 and 2

(d) Neither 1 nor 2

Solution: A

Page 23: INSTA Revision Plan 3.0 - 2020 INSTA Tests - INSIGHTSIAS...2020/08/04  · palm oil. Palm oil contributes 70% of vegetable oil import and is one of the cheapest oil due to high productivity

www.insightsonindia.com 21 INSTA Revision 3.0

INSIGHTSIAS SIMPLYFYING IAS EXAM PREPARATION

Indian Ocean Dipole effect

• However, it was discovered that just like ENSO was an event in the Pacific Ocean,

a similar seesaw ocean-atmosphere system in the Indian Ocean was also at play.

It was discovered in 1999 and named the Indian Ocean Dipole (IOD).

• The Indian Ocean Dipole (IOD) is defined by the difference in sea surface

temperature between two areas (or poles, hence a dipole) – a western pole in the

Arabian Sea (western Indian Ocean) and an eastern pole in the eastern Indian

Ocean south of Indonesia.

• IOD develops in the equatorial region of Indian Ocean from April to May

peaking in October.

• With a positive IOD wind over the Indian Ocean blow from east to west (from

Bay of Bengal towards Arabian Sea). This results in the Arabian Sea (western

Indian Ocean near African Coast) being much warmer and eastern Indian Ocean

around Indonesia becoming colder and dry.

• In the negative dipole year (negative IOD), reverse happens making Indonesia

much warmer and rainier.

• It was demonstrated that a positive IOD index often negated the effect of

ENSO, resulting in increased Monsoon rains in several ENSO years like the

1983, 1994 and 1997.

• Further, it was shown that the two poles of the IOD – the eastern pole (around

Indonesia) and the western pole (off the African coast) were independently and

cumulatively affecting the quantity of rains for the Monsoon in the Indian

subcontinent.

• Similar to ENSO, the atmospheric component of the IOD was later discovered

and named as Equatorial Indian Ocean Oscillation [EQUINOO][Oscillation of

warm water and atmospheric pressure between Bay of Bengal and Arabian Sea].

Impact on IOD on Cyclonogeneis in Northern Indian Ocean

• Positive IOD (Arabian Sea warmer than Bay of Bengal) results in more cyclones

than usual in Arabian Sea.

• Negative IOD results in stronger than usual cyclonogenesis (Formation of

Tropical Cyclones) in Bay of Bengal. Cyclonogenesis in Arabian Sea is

suppressed.

28. Gale crater, sometimes seen in news, is situated on the planet:

(a) Mercury

(b) Venus

(c) Jupiter

(d) Mars

Page 24: INSTA Revision Plan 3.0 - 2020 INSTA Tests - INSIGHTSIAS...2020/08/04  · palm oil. Palm oil contributes 70% of vegetable oil import and is one of the cheapest oil due to high productivity

www.insightsonindia.com 22 INSTA Revision 3.0

INSIGHTSIAS SIMPLYFYING IAS EXAM PREPARATION

Solution: D

Gale is a crater, and probable dry lake, on Mars. It is 154 km in diameter and

estimated to be about 3.5-3.8 billion years old. Scientists chose Gale Crater as the

landing site for Curiosity because it has many signs that water was present over

its history.

https://www.firstpost.com/tech/science/scientists-think-they-know-what-the-

source-of-methane-in-mars-atmosphere-could-be-6373891.html

29. Consider the following statements regarding El Niño and El Niño Modoki

1. The El Niño Modoki phenomenon is characterized by the anomalously warm

central equatorial Pacific flanked by anomalously cool regions in both west

and east.

2. Conventional El Niño is characterized by strong anomalous cooling in the

eastern equatorial Pacific.

Which of the statements given above is/are correct?

(a) 1 only

(b) 2 only

(c) Both 1 and 2

(d) Neither 1 nor 2

Solution: A

The El Niño Modoki

• El Niño Modoki is a coupled ocean-atmosphere phenomenon in the tropical

Pacific.

• It is different from another coupled phenomenon in the tropical Pacific namely,

El Niño.

• Conventional El Niño is characterized by strong anomalous warming in the

eastern equatorial Pacific.

• Whereas, El Niño Modoki is associated with strong anomalous warming in

the central tropical Pacific and cooling in the eastern and western tropical

Pacific.

El Niño Modoki Impacts

• The El Niño Modoki phenomenon is characterized by the anomalously warm

central equatorial Pacific flanked by anomalously cool regions in both west and

east.

Page 25: INSTA Revision Plan 3.0 - 2020 INSTA Tests - INSIGHTSIAS...2020/08/04  · palm oil. Palm oil contributes 70% of vegetable oil import and is one of the cheapest oil due to high productivity

www.insightsonindia.com 23 INSTA Revision 3.0

INSIGHTSIAS SIMPLYFYING IAS EXAM PREPARATION

• Such zonal gradients result in anomalous two-cell Walker Circulation over the

tropical Pacific, with a wet region in the central Pacific.

30. Consider the following statements regarding the Environment Pollution Control

Authority (EPCA):

1. It is mandated by the High Court of Delhi tasked with taking various

measures to tackle air pollution in the National Capital Region.

2. It was notified in 1998 by Environment Ministry under the Air (Prevention

and Control of Pollution) Act, 1981.

Which of the statements given above is/are correct?

(a) 1 only

(b) 2 only

(c) Both 1 and 2

(d) Neither 1 nor 2

Solution: D

EPCA comes out with parking management plan for Delhi. In report, agency flags

free parking on public land, multiplicity of agencies to be key cause of congestion

and parking menace.

About Environment Pollution Control Authority (EPCA):

• EPCA was constituted with the objective of ‘protecting and improving’ the quality

of the environment and ‘controlling environmental pollution’ in the National

Capital Region. The EPCA also assists the apex court in various environment-

related matters in the region.

• EPCA is Supreme Court mandated body tasked with taking various measures

to tackle air pollution in the National Capital Region. It was notified in 1998

by Environment Ministry under Environment Protection Act, 1986.

Composition:

• Besides the chairman, the EPCA has 14 members, some of whom are the

environment secretary of the National Capital Territory of Delhi (NCT),

chairperson of the New Delhi Municipal Council, transport commissioner of the

NCT, the commissioners of various municipal corporations of Delhi and

professors at IIT Delhi and Jawaharlal Nehru University.

Functions:

• To protect and improve quality of environment and prevent and control

environmental pollution in National Capital Region.

• To enforce Graded Response Action Plan (GRAP) in NCR as per the pollution

levels.

Page 26: INSTA Revision Plan 3.0 - 2020 INSTA Tests - INSIGHTSIAS...2020/08/04  · palm oil. Palm oil contributes 70% of vegetable oil import and is one of the cheapest oil due to high productivity

www.insightsonindia.com 24 INSTA Revision 3.0

INSIGHTSIAS SIMPLYFYING IAS EXAM PREPARATION

https://www.insightsonindia.com/2019/04/01/environment-pollution-prevention-

and-control-authority-epca/

31. Consider the following statements regarding Trade Winds

1. The trade winds are those blowing from the equatorial low-pressure belt

towards the sub-tropical high-pressure areas.

2. These are confined to a region between 30°N and 30°S throughout the

earth’s surface.

3. They flow as the north-eastern trades in the northern hemisphere.

Which of the statements given above is/are correct?

(a) 1 and 2 only

(b) 1 and 3 only

(c) 2 and 3 only

(d) 1, 2 and 3

Solution: C

Trade Winds

• The trade winds are those blowing from the sub-tropical high-pressure areas

towards the equatorial low-pressure belt.

• Therefore, these are confined to a region between 30°N and 30°S throughout

the earth’s surface.

• They flow as the north-eastern trades in the northern hemisphere and the

south-eastern trades in the southern hemisphere.

• This deflection in their ideally expected north-south direction is explained on

the basis of Coriolis force and Farrel’s law.

• Trade winds are descending and stable in areas of their origin (sub-tropical

high pressure belt), and as they reach the equator, they become humid and

warmer after picking up moisture on their way.

• The trade winds from two hemispheres meet at the equator, and due to

convergence they rise and cause heavy rainfall.

• The eastern parts of the trade winds associated with the cool ocean currents

are drier and more stable than the western parts of the ocean.

Page 27: INSTA Revision Plan 3.0 - 2020 INSTA Tests - INSIGHTSIAS...2020/08/04  · palm oil. Palm oil contributes 70% of vegetable oil import and is one of the cheapest oil due to high productivity

www.insightsonindia.com 25 INSTA Revision 3.0

INSIGHTSIAS SIMPLYFYING IAS EXAM PREPARATION

32. Consider the following statements regarding Rossby Waves

1. Rossby waves are natural phenomenon in the atmosphere and oceans due

to rotation of earth.

2. Rossby waves are formed when polar air moves toward the Equator while

tropical air is moving poleward.

Which of the statements given above is/are correct?

(a) 1 only

(b) 2 only

(c) Both 1 and 2

(d) Neither 1 nor 2

Solution: C

Rossby Waves

• The meandering jet streams are called Rossby Waves.

• Rossby waves are natural phenomenon in the atmosphere and oceans due to

rotation of earth.

• In planetary atmospheres, they are due to the variation in the Coriolis effect

(When temperature contrast is low, speed of jet stream is low, and Coriolis force

is weak leading to meandering) with latitude.

• Rossby waves are formed when polar air moves toward the Equator while

tropical air is moving poleward.

• The existence of these waves explains the low-pressure cells (cyclones) and

high-pressure cells (anticyclones).

33. Global Energy and CO2 Status Report, has been released by:

(a) United Nation Environment Program (UNEP)

(b) The United Nations Framework Convention on Climate Change (UNFCC)

(c) The International Energy Agency (IEA)

(d) International Association of Oil & Gas Producers

Solution: C

The International Energy Agency ‘s (IEA) Global Energy & CO2 Status Report

provides a snapshot of recent global trends and developments across fuels,

renewable sources, and energy efficiency and carbon emissions

Page 28: INSTA Revision Plan 3.0 - 2020 INSTA Tests - INSIGHTSIAS...2020/08/04  · palm oil. Palm oil contributes 70% of vegetable oil import and is one of the cheapest oil due to high productivity

www.insightsonindia.com 26 INSTA Revision 3.0

INSIGHTSIAS SIMPLYFYING IAS EXAM PREPARATION

http://www.iea.org/events/global-energy-co2-status-report

34. Consider the following statements regarding British Type Climate

1. The seasons are very distinct.

2. Rainfall occurs throughout the year with summer maxima.

3. The cool temperate western margins are under the influence of the

Westerlies all-round the year.

Which of the statements given above is/are correct?

(a) 1 and 2 only

(b) 1 and 3 only

(c) 2 and 3 only

(d) 1, 2 and 3

Solution: B

British Type Climate

• Westerlies come all the year round.

• There is a tendency towards an autumn or winter maximum of rainfall.

• Light snow falls in winter.

• Ports are never frozen but frosts do occur on cold nights.

• The seasons are very distinct.

• And the climate is very favorable for maximum human output.

British Type Climate or Cool Temperate Western Margin Climate or North-West

European Maritime Climate.

• The cool temperate western margins are under the influence of the Westerlies

all-round the year.

• They are the regions of frontal cyclonic activity [Temperate Cyclones].

• This type of climate is typical to Britain, hence the name ‘British Type’.

• Also called as North-West European Maritime Climate due to greater oceanic

influence.

British Type Climate

• Moderately warm summers and fairly mild winters.

• Rainfall occurs throughout the year with winter maxima.

Page 29: INSTA Revision Plan 3.0 - 2020 INSTA Tests - INSIGHTSIAS...2020/08/04  · palm oil. Palm oil contributes 70% of vegetable oil import and is one of the cheapest oil due to high productivity

www.insightsonindia.com 27 INSTA Revision 3.0

INSIGHTSIAS SIMPLYFYING IAS EXAM PREPARATION

35. Consider the following statements regarding the Outer Space Treaty, 1967:

1. States shall be responsible for national space activities carried out by

governmental entities only.

2. It prohibits all kinds of weapons to be placed in outer space.

3. States shall be liable for damage caused by their space objects

Which of the statements given above is/are correct?

(a) 1 and 2 only

(b) 3 only

(c) 2 and 3 only

(d) 1 only

Solution: B

The Outer Space Treaty was considered by the Legal Subcommittee in 1966 and

agreement was reached in the General Assembly in the same year ( resolution 2222

(XXI)). The Treaty was largely based on the Declaration of Legal Principles

Governing the Activities of States in the Exploration and Use of Outer Space,

which had been adopted by the General Assembly in its resolution 1962 (XVIII) in

1963, but added a few new provisions. The Treaty was opened for signature by the

three depository Governments (the Russian Federation, the United Kingdom and

the United States of America) in January 1967, and it entered into force in October

1967.

The Outer Space Treaty provides the basic framework on international space law,

including the following principles:

• the exploration and use of outer space shall be carried out for the benefit and in

the interests of all countries and shall be the province of all mankind;

• outer space shall be free for exploration and use by all States;

• outer space is not subject to national appropriation by claim of sovereignty, by

means of use or occupation, or by any other means;

o States shall not place nuclear weapons or other weapons of mass

destruction in orbit or on celestial bodies or station them in outer space

in any other manner;

• the Moon and other celestial bodies shall be used exclusively for peaceful

purposes;

• astronauts shall be regarded as the envoys of mankind;

o States shall be responsible for national space activities whether carried

out by governmental or non-governmental entities;

o States shall be liable for damage caused by their space objects; and

Page 30: INSTA Revision Plan 3.0 - 2020 INSTA Tests - INSIGHTSIAS...2020/08/04  · palm oil. Palm oil contributes 70% of vegetable oil import and is one of the cheapest oil due to high productivity

www.insightsonindia.com 28 INSTA Revision 3.0

INSIGHTSIAS SIMPLYFYING IAS EXAM PREPARATION

o States shall avoid harmful contamination of space and celestial bodies.

• The Outer Space Treaty prohibits only weapons of mass destruction in outer

space, not ordinary weapons.

• As of February 2019, 108 countries are parties to the treaty, while another 23

have signed the treaty but have not completed ratification.

https://www.thehindu.com/opinion/editorial/outer-clarity/article26737897.ece

https://www.unoosa.org/oosa/en/ourwork/spacelaw/treaties/introouterspacetrea

ty.html

36. Consider the following statements regarding rainfall in India

1. Monsoons winds flowing in Rajasthan and Gujarat are not obstructed by

any orographic barrier and hence these regions receive no rainfall.

2. Cherrapunji and Mawsynram receive abnormally high rainfall due to

funneling effect followed by orographic upliftment.

Which of the statements given above is/are correct?

(a) 1 only

(b) 2 only

(c) Both 1 and 2

(d) Neither 1 nor 2

Solution: C

Why no significant rainfall in Gujarat and Rajasthan? Explain the formation of

Thar Desert?

• Monsoons winds flowing in Rajasthan and Gujarat are not obstructed by any

orographic barrier and hence these regions receive no rainfall.

[Monsoon winds blow almost parallel to Aravalis and hence there is no orographic

rainfall].

[No convection cell or vertical wind movements arise in Rajasthan and Gujarat:

Monsoon winds blow towards low pressure cells in Tibet and hence only horizontal

wind movements exist in Gujarat and Rajasthan]

[Sub-tropical high pressure belt: In winter the region experiences strong divergence

because of the STJ – Sub-Tropical Jet.]

How come Cherrapunji and Mawsynram receive abnormally high rainfall?

• Mawsynram and Cherrapunji are the wettest places on earth with mean annual

rainfall over 1000 cm.

Page 31: INSTA Revision Plan 3.0 - 2020 INSTA Tests - INSIGHTSIAS...2020/08/04  · palm oil. Palm oil contributes 70% of vegetable oil import and is one of the cheapest oil due to high productivity

www.insightsonindia.com 29 INSTA Revision 3.0

INSIGHTSIAS SIMPLYFYING IAS EXAM PREPARATION

• Copious rainfall in these places is due to funneling effect followed by orographic

upliftment. [Funneling effect = clouds are channeled into a narrow region

between mountains and hence the cloud density is extraordinary]

37. Consider the following statements regarding Easterly Jet Stream

1. This jet stream is responsible for bringing western disturbances from the

Mediterranean region in to the Indian sub-continent.

2. This helps in the sudden onset of the south-west monsoons.

Which of the statements given above is/are correct?

(a) 1 only

(b) 2 only

(c) Both 1 and 2

(d) Neither 1 nor 2

Solution: B

Westerly Jet Stream

• Westerly jet stream blows at a very high-speed during winter over the sub-

tropical zone.

• Southern branch of the jet stream exercises a significant influence on the winter

weather conditions in India.

• This jet stream is responsible for bringing western disturbances from the

Mediterranean region in to the Indian sub-continent.

• Winter rain and heat storms in north-western plains and occasional heavy

snowfall in hilly regions are caused by these disturbances.

• These are generally followed by cold waves in the whole of northern plains.

Easterly Jet Stream

• Reversal in upper air circulation takes place in summer due to the apparent

shift of the sun’s vertical rays in the northern hemisphere.

• The westerly jet stream is replaced by the easterly jet stream which owes its

origin to the heating of the Tibet plateau.

• This helps in the sudden onset of the south-west monsoons.

Page 32: INSTA Revision Plan 3.0 - 2020 INSTA Tests - INSIGHTSIAS...2020/08/04  · palm oil. Palm oil contributes 70% of vegetable oil import and is one of the cheapest oil due to high productivity

www.insightsonindia.com 30 INSTA Revision 3.0

INSIGHTSIAS SIMPLYFYING IAS EXAM PREPARATION

38. Consider the following statements regarding the Pradhan Mantri Sahaj Bijli Har

Ghar Yojana (Saubhagya):

1. The objective of the scheme is to provide energy access to all by last mile

connectivity.

2. The beneficiaries for free electricity connections would be identified using

Socio Economic and Caste Census (SECC) 2011 data.

3. The Rural Electrification Corporation Limited (REC) will remain the nodal

agency for the operationalisation of the scheme throughout the country.

Which of the statements given above is/are correct?

(a) 1 and 2 only

(b) 2 and 3 only

(c) 1 and 3 only

(d) 1, 2 and 3

Solution: D

Pradhan Mantri Sahaj Bijli Har Ghar Yojana –“Saubhagya” is a scheme to ensure

electrification of all willing households in the country in rural as well as urban

areas.

The objective of the ‘Saubhagya’ is to provide energy access to all by last mile

connectivity and electricity connections to all remaining un-electrified households in

rural as well as urban areas to achieve universal household electrification in the

country.

The beneficiaries for free electricity connections would be identified using Socio

Economic and Caste Census (SECC) 2011 data. However, un-electrified

households not covered under the SECC data would also be provided electricity

connections under the scheme on payment of Rs. 500 which shall be recovered by

DISCOMs in 10 instalments through electricity bill.

The solar power packs of 200 to 300 Wp with battery bank for un-electrified

households located in remote and inaccessible areas, comprises of Five LED lights,

One DC fan, One DC power plug. It also includes the Repair and Maintenance (R&M)

for 5 years.

The Rural Electrification Corporation Limited (REC) will remain the nodal agency

for the operationalisation of the scheme throughout the country.

https://www.thehindu.com/opinion/op-ed/how-to-achieve-24×7-power-for-

all/article26714432.ece

https://vikaspedia.in/energy/policy-support/pradhan-mantri-sahaj-bijli-har-ghar-

yojana

Page 33: INSTA Revision Plan 3.0 - 2020 INSTA Tests - INSIGHTSIAS...2020/08/04  · palm oil. Palm oil contributes 70% of vegetable oil import and is one of the cheapest oil due to high productivity

www.insightsonindia.com 31 INSTA Revision 3.0

INSIGHTSIAS SIMPLYFYING IAS EXAM PREPARATION

39. Consider the following statements regarding Red Soils

1. These soils mostly occur in the regions of low rainfall.

2. They are poor in lime, magnesia, phosphates, nitrogen and humus.

3. They are basic mainly due to the nature of the parent rocks.

Which of the statements given above is/are correct?

(a) 1 and 2 only

(b) 1 and 3 only

(c) 2 and 3 only

(d) 1, 2 and 3

Solution: A

Red Soils

• Red soils along with its minor groups form the largest soil group of India.

• The main parent rocks are crystalline and metamorphic rocks like acid

granites, gneisses and quartzites.

Characteristics of Red Soils

• The texture of these soils can vary from sand to clay, the majority being loams.

• On the uplands, the red soils are poor, gravelly, and porous. But in the lower

areas they are rich, deep dark and fertile.

Chemical Composition of Red Soils

• They are acidic mainly due to the nature of the parent rocks. The alkali content

is fair.

• They are poor in lime, magnesia, phosphates, nitrogen and humus.

• They are fairly rich in potash and potassium.

Distribution of Red Soils

• These soils mostly occur in the regions of low rainfall.

• They occupy about 3.5 lakh sq km (10.6 per cent) of the total area of the country.

40. Consider the following statements regarding the Indian Agriculture:

1. India is the largest producer of spices.

2. Agricultural export constitutes around 10 per cent of the country’s exports.

3. India is the second largest producer of sugar.

Page 34: INSTA Revision Plan 3.0 - 2020 INSTA Tests - INSIGHTSIAS...2020/08/04  · palm oil. Palm oil contributes 70% of vegetable oil import and is one of the cheapest oil due to high productivity

www.insightsonindia.com 32 INSTA Revision 3.0

INSIGHTSIAS SIMPLYFYING IAS EXAM PREPARATION

Which of the statements given above is/are correct?

(a) 1 and 2 only

(b) 2 and 3 only

(c) 1 and 3 only

(d) 1, 2 and 3

Solution: D

Agriculture, along with fisheries and forestry, is one of the largest contributors

to the Gross Domestic Product.

Gross Value Added by agriculture, forestry and fishing is estimated at Rs 17.67

trillion (US$ 274.23 billion) in FY18.

As per estimates by the Central Statistics Office (CSO), the share of agriculture and

allied sectors (including agriculture, livestock, forestry and fishery) was 15.35 per

cent of the Gross Value Added (GVA) during 2015-16.

India is the largest producer, consumer and exporter of spices and spice

products.

Agricultural export constitutes 10 per cent of the country’s exports and is the fourth-

largest exported principal commodity.

India, the second-largest producer of sugar, accounts for 14 per cent of the global

output. It is the sixth-largest exporter of sugar, accounting for 2.76 per cent of the

global exports. India is the leading producer of wheat, rice, cotton, sugarcane &

vegetables.

https://www.financialexpress.com/opinion/agriculture-needs-a-shift-in-focus-ten-

ways-india-can-do-it/1542175/

41. Consider the following statements regarding Forest/Mountain Soils

1. The formation of these soils is mainly governed by the characteristic

deposition of organic matter derived from forests.

2. Their character changes with parent rocks, ground-configuration and

climate, hence they differ greatly even if they occur in close proximity to one

another.

3. They are rich in potash, phosphorus and lime.

Which of the statements given above is/are correct?

(a) 1 and 2 only

(b) 1 and 3 only

(c) 2 and 3 only

Page 35: INSTA Revision Plan 3.0 - 2020 INSTA Tests - INSIGHTSIAS...2020/08/04  · palm oil. Palm oil contributes 70% of vegetable oil import and is one of the cheapest oil due to high productivity

www.insightsonindia.com 33 INSTA Revision 3.0

INSIGHTSIAS SIMPLYFYING IAS EXAM PREPARATION

(d) 1, 2 and 3

Solution: A

Forest – Mountain Soils

• These soils occupy about 2.85 lakh sq km or 8.67% of the total land area of India.

• They are mainly heterogeneous soils found on the hill slopes covered by forests.

• The formation of these soils is mainly governed by the characteristic deposition

of organic matter derived from forests and their character changes with parent

rocks, ground-configuration and climate.

• Consequently, they differ greatly even if they occur in close proximity to one

another.

Distribution of Forest – Mountain Soils

• In the Himalayan region, such soils are mainly found in valleys, less steep and

north facing slopes. The south facing slopes are very steep and exposed to

denudation and hence do not support soil formation.

• Forest soils occur in Western and Eastern Ghats also.

Chemical properties of Forest – Mountain Soils

• The forest soils are very rich in humus.

• They are deficient in potash, phosphorus and lime.

• They require good deal of fertilizers for high yields.

42. Which of the following measures control soil erosion in India?

1. Ban on shifting cultivation

2. Controlled grazing

3. Mixed cropping

Select the correct answer using the code given below

(a) 2 only

(b) 1 and 3 only

(c) 1, 2 and 3

(d) 2 and 3 only

Solution: C

Page 36: INSTA Revision Plan 3.0 - 2020 INSTA Tests - INSIGHTSIAS...2020/08/04  · palm oil. Palm oil contributes 70% of vegetable oil import and is one of the cheapest oil due to high productivity

www.insightsonindia.com 34 INSTA Revision 3.0

INSIGHTSIAS SIMPLYFYING IAS EXAM PREPARATION

Soil conservation is the prevention of loss of the top most layer of the soil from

erosion or prevention of reduced fertility caused by over usage, acidification,

salinization or other chemical soil contamination.

Contour tillage: Here the fields are ploughed along the contours and not along the

hill slope. This type of plantation is very effective as it leads to the formation of ridges

and furrows against the direction of flow and decreases the velocity of water.

Contour Bunding: Here first the plants and bushes in the plot of shifting cultivation

is cut down. Leaves are slashed down and left on the surface of the soil to get dry.

After the decomposition of the leaves and various other plant parts, the dry materials

are bundled and arranged in contour lines.

Some other measures to control soil erosion are: construction of check dams, ban

on shifting cultivation, controlled grazing, afforestation, mixed cropping and

mixed farming etc.

Planting species of plants that can restore the ecological balance of an eroded area

is also necessary as rehabilitation of the damaged soil is as much important as its

prevention from getting eroded.

43. Chandaka-Dampara Wildlife Sanctuary, is located in:

(a) West Bengal

(b) Odisha

(c) Manipur

(d) Madhya Pradesh

Solution: B

Chandaka Sanctuary is a wildlife reserve located in north western fringe of

Bhubaneswar in the Indian state of Odisha. Chandaka –Dampara Wildlife

Sanctuary comprises major forest area i.e. notified forest blocks of Chandaka

Wildlife Division.

Four decades later, bamboo rice shows up in Odisha. The rare variety, which was

last harvested in Chandaka-Dampara Wildlife Sanctuary in 1979, grows twice or

thrice in a century.

What is Bamboo Rice?

Bamboo rice is special rice that is grown out of a dying bamboo shoot. When the

bamboo shoot breathes its last, it flowers into a rare variety of rice seeds, which are

known as bamboo rice.

44. Which of the following forces/factors affects the velocity and direction of Wind?

1. Pressure gradient force

Page 37: INSTA Revision Plan 3.0 - 2020 INSTA Tests - INSIGHTSIAS...2020/08/04  · palm oil. Palm oil contributes 70% of vegetable oil import and is one of the cheapest oil due to high productivity

www.insightsonindia.com 35 INSTA Revision 3.0

INSIGHTSIAS SIMPLYFYING IAS EXAM PREPARATION

2. Coriolis force

3. Gravitational force

Select the correct answer using the code given below:

(a) 1, 2 and 3

(b) 2 and 3 only

(c) 1 and 2 only

(d) 1 and 3 only

Solution: A

Forces Affecting the Velocity and Direction of Wind

• The air is set in motion due to the differences in atmospheric pressure. The

air in motion is called wind. The wind blows from high pressure to low

pressure. The wind at the surface experiences friction. In addition, rotation of

the earth also affects the wind movement.

• The force exerted by the rotation of the earth is known as the Coriolis force.

Thus, the horizontal winds near the earth surface respond to the combined

effect of three forces – the pressure gradient force, the frictional force and

the Coriolis force. In addition, the gravitational force acts downward.

45. Bold Kurukshetra – 2019, is a joint military exercise between:

(a) India – Bangladesh

(b) India – Bolivia

(c) India – Singapore

(d) India – Afghanistan

Solution: C

The 12th edition of joint military exercise between India and Singapore, BOLD

KURUKSHETRA 2019, culminated on 11 April 2019 after an impressive closing

ceremony held at Babina Military Station. The four day long joint training focused

on developing interoperability and conduct of joint tactical operations in mechanised

warfare.

Page 38: INSTA Revision Plan 3.0 - 2020 INSTA Tests - INSIGHTSIAS...2020/08/04  · palm oil. Palm oil contributes 70% of vegetable oil import and is one of the cheapest oil due to high productivity

www.insightsonindia.com 36 INSTA Revision 3.0

INSIGHTSIAS SIMPLYFYING IAS EXAM PREPARATION

46. Consider the following statements regarding Western disturbances in Northern

Plains

1. These are low-pressure systems, originate over the Mediterranean Sea and

Western Asia

2. They are of immense importance for the cultivation of ‘rabi’ crops.

Which of the statements given above is/are correct?

(a) 1 only

(b) 2 only

(c) Both 1 and 2

(d) Neither 1 nor 2

Solution: C

A characteristic feature of the cold weather season over the northern plains is the

inflow of cyclonic disturbances from the west and the northwest. These low-pressure

systems, originate over the Mediterranean Sea and western Asia and move into India,

along with the westerly flow.

They cause the much-needed winter rains over the plains and snowfall in the

mountains. Although the total amount of winter rainfall locally known as

‘mahawat’ is small, they are of immense importance for the cultivation of ‘rabi’

crops.

47. Consider the following statements regarding El-Nino and La-Nina

1. El Nino and La Nina episodes typically last nine to 12 months.

2. El Nino and La Nina events occur on average every two to seven years.

Which of the statements given above is/are not correct?

(a) 1 only

(b) 2 only

(c) Both 1 and 2

(d) Neither 1 nor 2

Solution: D

La Niña is sometimes referred to as the cold phase of ENSO and El Niño as the warm phase of ENSO. These deviations from normal surface temperatures can have

large-scale impacts not only on ocean processes, but also on global weather and

climate.

Page 39: INSTA Revision Plan 3.0 - 2020 INSTA Tests - INSIGHTSIAS...2020/08/04  · palm oil. Palm oil contributes 70% of vegetable oil import and is one of the cheapest oil due to high productivity

www.insightsonindia.com 37 INSTA Revision 3.0

INSIGHTSIAS SIMPLYFYING IAS EXAM PREPARATION

El Niño and La Niña episodes typically last 9 to 12 months, but some prolonged

events may last for years. While their frequency can be quite irregular, El Niño and

La Niña events occur on average every two to seven years. Typically, El Niño

occurs more frequently than La Niña.

https://oceanservice.noaa.gov/facts/ninonina.html

48. Consider the following statements regarding the International Maritime

Organisation (IMO):

1. It is a specialized agency of the United Nations.

2. It is the global standard-setting authority for the safety, security and

environmental performance of international shipping.

Which of the statements given above is/are correct?

(a) 1 only

(b) 2 only

(c) Both 1 and 2

(d) Neither 1 nor 2

Solution: C

As a specialized agency of the United Nations, IMO is the global standard-setting

authority for the safety, security and environmental performance of international

shipping. Its main role is to create a regulatory framework for the shipping industry

that is fair and effective, universally adopted and universally implemented.

In other words, its role is to create a level playing-field so that ship operators cannot

address their financial issues by simply cutting corners and compromising on safety,

security and environmental performance. This approach also encourages

innovation and efficiency.

https://www.thehindubusinessline.com/economy/logistics/imos-new-rule-on-

electronic-information-exchange-between-ships-and-ports-comes-into-

force/article26771859.ece

http://www.imo.org/en/About/Pages/Default.aspx

49. Consider the following statements regarding the Temperate Cyclone

1. Temperate cyclone is largely of thermal origin.

2. Temperate cyclone can form both on land as well as seas.

3. Temperate cyclones typically won’t last for more than one week.

Which of the statements given above is/are correct?

Page 40: INSTA Revision Plan 3.0 - 2020 INSTA Tests - INSIGHTSIAS...2020/08/04  · palm oil. Palm oil contributes 70% of vegetable oil import and is one of the cheapest oil due to high productivity

www.insightsonindia.com 38 INSTA Revision 3.0

INSIGHTSIAS SIMPLYFYING IAS EXAM PREPARATION

(a) 2 only

(b) 1 only

(c) 1 and 3 only

(d) 1 and 2 only

Solution: A

A tropical cyclone is a rapidly rotating storm system characterized by a low-pressure center, a closed low-level atmospheric circulation, strong winds, and a

spiral arrangement of thunderstorms that produce heavy rain or squalls.

Tropical cyclone is largely of thermal origin. Tropic cyclone can form only on seas.

Temperate Cyclones or Extra tropical cyclones, sometimes called mid-latitude cyclones or wave cyclones, are low-pressure areas which, along with the anticyclones of high-pressure areas, drive the weather over much of the Earth.

Temperate cyclone can form both on land as well as seas. Temperate cyclones

typically last for 2-3 weeks.

50. Recently, on which of the following rivers ‘Maitri Bridge’ was constructed by

Indian Army?

(a) Jhelum

(b) Ravi

(c) Indus

(d) Brahmaputra

Solution: C

Indian Army built the longest suspension bridge over Indus River in Leh in just

40 days, making a record of its own. Named Maitri Bridge, the construction of the

bridge was undertaken by the combat engineers of ‘Sahas aur Yogyata’ regiment of

‘Fire and Fury Corps’ of the Indian Army. The bridge is built using around 500

tonnes of bridge equipment and construction material.

The 260-feet long bridge is likely to give a major boost to connectivity in remote

areas in Ladakh. The cable suspension bridge that has been opened for the public

was constructed to help the residents of Choglamsar, Stok and Chuchot villages

https://www.businesstoday.in/current/economy-politics/longest-suspension-

bridge-over-indus-river-opens-to-public-heres-all-you-need-to-

know/story/333520.html

Page 41: INSTA Revision Plan 3.0 - 2020 INSTA Tests - INSIGHTSIAS...2020/08/04  · palm oil. Palm oil contributes 70% of vegetable oil import and is one of the cheapest oil due to high productivity

www.insightsonindia.com 39 INSTA Revision 3.0

INSIGHTSIAS SIMPLYFYING IAS EXAM PREPARATION

DAY – 15

51. Consider the following statements regarding drainage patterns

1. In a concordant drainage pattern, the river follows its initial path

irrespective of the changes in topography.

2. In a discordant drainage pattern, the path of the river is highly dependent

on the slope of the river and topography.

Which of the statements given above is/are correct?

(a) 1 only

(b) 2 only

(c) Both 1 and 2

(d) Neither 1 nor 2

Solution: D

Concordant drainage patterns

• A drainage pattern is described as concordant if it correlates to the topology and

geology of the area.

• In simple words, in a concordant drainage pattern, the path of the river is highly

dependent on the slope of the river and topography.

• Concordant drainage patterns are the most commonly found drainage patterns

and are classified into many consequent, subsequent, obsequent and resequent.

Discordant or Insequent drainage patterns

• A drainage pattern is described as discordant if it does not correlate to the

topology (surface relief features) and geology of the area.

• In simple words, in a discordant drainage pattern, the river follows its initial path

irrespective of the changes in topography.

• Discordant drainage patterns are classified into two main types: antecedent and

superimposed.

• Usually, rivers in both these drainage types flow through a highly sloping

surface.

52. Consider the following statements regarding Damodar River

1. It rises in the Amarkantak Plateau.

2. It flows through a rift valley.

Page 42: INSTA Revision Plan 3.0 - 2020 INSTA Tests - INSIGHTSIAS...2020/08/04  · palm oil. Palm oil contributes 70% of vegetable oil import and is one of the cheapest oil due to high productivity

www.insightsonindia.com 40 INSTA Revision 3.0

INSIGHTSIAS SIMPLYFYING IAS EXAM PREPARATION

3. It is called as ‘Sorrow of Bengal’.

Which of the statements given above is/are correct?

(a) 1 and 2 only

(b) 1 and 3 only

(c) 2 and 3 only

(d) 1, 2 and 3

Solution: C

Son River

• The Son River rises in the Amarkantak Plateau.

• Its source is close to the origin of the Narmada.

• It passes along the Kaimur Range.

• It joins the Ganga near Danapur in Patna district of Bihar.

• It flows for a distance of 784 km from its source.

• The important tributaries of the Son are the Johilla, the Gopat, the Rihand, the

Kanhar and the North Koel. Almost all the tributaries join it on its right bank.

Damodar river

• The Damodar river rises in the hills of the Chotanagpur plateau and flows

through a rift valley.

• Rich in mineral resources, the valley is home to large-scale mining and

industrial activity.

• It has a number of tributaries and subtributaries, such as Barakar, Konar,

Bokaro, Haharo, etc.

• The Barakar is the most important tributary of the Damodar.

• Several dams have been constructed in the valley, for the generation of

hydroelectric power. The valley is called “the Ruhr of India”.

• The first dam was built across the Barakar River, a tributary of the Damodar

river.

• It used to cause devastating floods as a result of which it earned the name

‘Sorrow of Bengal’. Now the river is tamed by constructing numerous dams.

Page 43: INSTA Revision Plan 3.0 - 2020 INSTA Tests - INSIGHTSIAS...2020/08/04  · palm oil. Palm oil contributes 70% of vegetable oil import and is one of the cheapest oil due to high productivity

www.insightsonindia.com 41 INSTA Revision 3.0

INSIGHTSIAS SIMPLYFYING IAS EXAM PREPARATION

53. Hala’ib triangle, a disputed territory, sometimes seen in news, is located on the

coast of:

(a) Persian Gulf

(b) Gulf of Oman

(c) Red Sea

(d) South China Sea

Solution: C

The Halayib Triangle, also known as the Halayeb Triangle, is an area of land

measuring 20,580 square kilometres located on the Northeast African coast of the

Red Sea.

It is a disputed territory between Sudan and Egypt.

54. Consider the following statements regarding Peninsular River System

1. Peninsula rivers are much older than the Himalayan rivers and have

reached mature stage.

2. The peninsular drainage is mainly Discordant.

3. The main water divide in peninsular rivers is formed by the Western Ghats.

Which of the statements given above is/are correct?

(a) 1 and 2 only

(b) 1 and 3 only

(c) 2 and 3 only

(d) 1, 2 and 3

Solution: B

Peninsular River System or Peninsular Drainage

• Peninsula rivers are much older than the Himalayan rivers {Discordant}.

• The peninsular drainage is mainly Concordant except for few rivers in the upper

peninsula region.

• They are non-perennial rivers with a maximum discharge in the rainy season.

• The peninsular rivers have reached mature stage {Fluvial Landforms} and have

almost reached their base level. [Vertical downcutting is negligible].

Page 44: INSTA Revision Plan 3.0 - 2020 INSTA Tests - INSIGHTSIAS...2020/08/04  · palm oil. Palm oil contributes 70% of vegetable oil import and is one of the cheapest oil due to high productivity

www.insightsonindia.com 42 INSTA Revision 3.0

INSIGHTSIAS SIMPLYFYING IAS EXAM PREPARATION

• The rivers are characterized by broad and shallow valleys.

• The river banks have gentle slopes except for a limited tract where faulting forms

steep sides.

• The main water divide in peninsular rivers is formed by the Western Ghats,

which run from north to south close to the western coast.

• The velocity of water in the rivers and the load carrying capacity of the streams

is low due to low gradient.

• Most of the major rivers of the peninsula such as the Mahanadi, the Godavari,

the Krishna and the Cauvery flow eastwards and drain into the Bay of Bengal.

These rivers make deltas at their mouths.

• But the west flowing rivers of Narmada and Tapi as well as those originating

from the Western Ghats and falling in the Arabian Sea form estuaries in place

of deltas.

55. Consider the following pairs of Harvest festivals in India and states they are

associated with:

Harvest Festivals States 1. Vishu - Maharashtra 2. Rongali Bihu - Odisha 3. Puthandu Pirappu - Goa

Which of the pairs given above is/are correctly matched?

(a) 1 and 3 only

(b) 2 only

(c) 1, 2 and 3

(d) None

Solution: D

Vishu is a Hindu festival celebrated in the Indian state of Kerala, Tulu Nadu region

in Karnataka, Mahe district of Union Territory of Pondicherry, neighbouring areas

of Tamil Nadu and their diaspora communities. The festival marks the first day of

Medam, the ninth month in the solar calendar followed in Kerala.

Bohag Bihu or Rongali Bihu also called Xaat Bihu is a festival celebrated in the

Indian state of Assam and other parts of northeastern India, and marks the

beginning of the Assamese New Year. It usually falls in the 2nd week of April,

historically signifying the time of harvest.

Puthandu, also known as Puthuvarusham or Tamil New Year, is the first day of

year on the Tamil calendar and traditionally celebrated as a festival. The festival

date is set with the solar cycle of the lunisolar Hindu calendar, as the first day of the

Tamil month Chithirai.

Page 45: INSTA Revision Plan 3.0 - 2020 INSTA Tests - INSIGHTSIAS...2020/08/04  · palm oil. Palm oil contributes 70% of vegetable oil import and is one of the cheapest oil due to high productivity

www.insightsonindia.com 43 INSTA Revision 3.0

INSIGHTSIAS SIMPLYFYING IAS EXAM PREPARATION

56. Consider the following statements regarding Jhelum River

1. The Jhelum originates from near the Bara Lacha Pass in the Lahul-Spiti

part of the Zaskar Range.

2. It flows northwards into Wular Lake.

Which of the statements given above is/are correct?

(a) 1 only

(b) 2 only

(c) Both 1 and 2

(d) Neither 1 nor 2

Solution: B

Jhelum River

• The Jhelum has its source in a spring at Verinag in the south-eastern part of

the Kashmir Valley.

• It flows northwards into Wular Lake (north-western part of Kashmir Valley).

From Wular Lake, it changes its course southwards. At Baramulla the river

enters a gorge in the hills.

• The river forms steep-sided narrow gorge through Pir Panjal Range below

Baramula.

• At Muzaffarabad, the river takes a sharp hairpin bend southward.

• Thereafter, it forms the India-Pakistan boundary for 170 km and emerges at the

Potwar Plateau near Mirpur.

• After flowing through the spurs of the Salt Range it debouches (emerge from a

confined space into a wide, open area) on the plains near the city of Jhelum.

• It joins the Chenab at Trimmu.

• The river is navigable for about 160 km out of a total length of 724 km.

Chenab River

• The Chenab originates from near the Bara Lacha Pass in the Lahul-Spiti part

of the Zaskar Range.

57. Which of the following are left bank tributaries of The Ganga River?

1. Ramganga River

2. Yamuna River

Page 46: INSTA Revision Plan 3.0 - 2020 INSTA Tests - INSIGHTSIAS...2020/08/04  · palm oil. Palm oil contributes 70% of vegetable oil import and is one of the cheapest oil due to high productivity

www.insightsonindia.com 44 INSTA Revision 3.0

INSIGHTSIAS SIMPLYFYING IAS EXAM PREPARATION

3. Damodar River

4. Gandak River

Select the correct answer using the code given below:

(a) 1 and 2 only

(b) 1 and 4 only

(c) 1, 2 and 4 only

(d) 1, 2, 3 and 4

Solution: B

Left Bank Tributaries of the Ganga River

• Ramganga River

• Ghaghra River

• Kali River

• Gandak River

• Burhi Gandak

• Kosi River

Right Bank Tributaries of the Ganga

• Yamuna River

• Damodar River

58. Consider the following statements regarding the Drugs Technical Advisory

Board (DTAB):

1. It is a non-statutory body, set up by executive resolution.

2. It is part of Central Drugs Standard Control Organization (CDSCO).

Which of the statements given above is/are correct?

(a) 1 only

(b) 2 only

(c) Both 1 and 2

(d) Neither 1 nor 2

Solution: B

Page 47: INSTA Revision Plan 3.0 - 2020 INSTA Tests - INSIGHTSIAS...2020/08/04  · palm oil. Palm oil contributes 70% of vegetable oil import and is one of the cheapest oil due to high productivity

www.insightsonindia.com 45 INSTA Revision 3.0

INSIGHTSIAS SIMPLYFYING IAS EXAM PREPARATION

DTAB is highest statutory decision-making body on technical matters related to

drugs in the country. It is constituted as per the Drugs and Cosmetics Act, 1940.

It is part of Central Drugs Standard Control Organization (CDSCO) in the Ministry

of Health and Family Welfare.

https://www.thehindu.com/sci-tech/health/government-notifies-medical-devices-

as-drugs-industry-cautious/article30812704.ece

59. Consider the following statements regarding Glaciers

1. The Siachen Glacier in Nubra valley is the largest glacier outside the polar

and the sub-polar regions.

2. The glaciers of the Pir Panjal Range are less numerous and smaller in size

as compared to those of the Karakoram Range.

3. Gangotri Glacier is located in Uttarkashi, Uttarakhand.

Which of the statements given above is/are correct?

(a) 1 and 2 only

(b) 1 and 3 only

(c) 2 and 3 only

(d) 1, 2 and 3

Solution: D

Glaciers of the Karakoram Range

• Maximum development of glaciers occurs in the Karakoram range.

• Some of the largest glaciers outside the polar and sub-polar regions are found in

this range. The southern side of this range has many gigantic glaciers.

• The 75 km long Siachen Glacier in Nubra valley has the distinction of being the

largest glacier outside the polar and the sub-polar regions.

• The second largest is the 74 km long Fedchenko Glacier (Pamirs)

• Third largest is the Hispar Glacier. It is 62 km long and occupies a tributary of

the Hunza River.

Glaciers of the Pir Panjal Range

• The glaciers of the Pir Panjal Range are less numerous and smaller in size as

compared to those of the Karakoram Range.

• The longest Sonapani Glacier in the Chandra Valley of Lahul and Spiti region is

only 15 km long.

Page 48: INSTA Revision Plan 3.0 - 2020 INSTA Tests - INSIGHTSIAS...2020/08/04  · palm oil. Palm oil contributes 70% of vegetable oil import and is one of the cheapest oil due to high productivity

www.insightsonindia.com 46 INSTA Revision 3.0

INSIGHTSIAS SIMPLYFYING IAS EXAM PREPARATION

• Gangotri Glacier: located in Uttarkashi, Uttarakhand

• Milam Glacier: located in Uttarkashi, Uttarakhand

• Pindari glacier Nanda devi, Uttarakhand

• Zemu Glacier Largest glacier in the Eastern Himalaya Located on Kanchenjunga

peak, Sikkim

• Baltoro Glacier J & K – Karakoram range

• Chogolungma glacier J & K – Karakoram range

• Diamir Glacier J & K – Karakoram range

• Siachin Glacier LOC between India & POK; Close to China border; – Karakoram

Range

60. Consider the following statements regarding the Asia Pacific Broadcasting

Union (ABU):

1. It was established as a non-profit, non-governmental, non- political,

professional association.

2. It stipulates that there may be only five Full Members in any one country.

3. It is the biggest broadcasting union in the world.

Which of the statements given above is/are correct?

(a) 1 and 2 only

(b) 2 and 3 only

(c) 1 and 3 only

(d) 1, 2 and 3

Solution: C

The Asia-Pacific Broadcasting Union (ABU) is the biggest broadcasting union in the

world. The Union was established in 1964 as a non-profit, non-governmental, non-

political, professional association with mandate to assist the development of

broadcasting in the region. ABU promotes the collective interests of television and

radio broadcasters as well as key industry players and facilitate regional and

international media co-operation.

ABU is a member of the World Broadcasters’ Union and works closely with the

other regional broadcasting unions on matters of common concern such as reserving

frequencies for broadcasters, harmonisation of operating and technical broadcasting

standards and systems and finalising the Broadcasting Treaty.

Page 49: INSTA Revision Plan 3.0 - 2020 INSTA Tests - INSIGHTSIAS...2020/08/04  · palm oil. Palm oil contributes 70% of vegetable oil import and is one of the cheapest oil due to high productivity

www.insightsonindia.com 47 INSTA Revision 3.0

INSIGHTSIAS SIMPLYFYING IAS EXAM PREPARATION

Membership is open to national broadcasters and national broadcasting

organisations. There are only two Full members in each country so that there is a

balance of voting rights across the region.

The ABU’s “board of directors” is called the Administrative Council. It consists of 17

of its Full and Additional Full members, elected for a three year term.

http://www.newsonair.com/News?title=ABU-Media-Summit-on-Climate-Action-

and-Disaster-Preparedness-underway-in-Nepal&id=362714

61. Consider the following statements regarding Biomass

1. The energy from biomass is released on burning or breaking the chemical

bonds of organic molecules formed during photosynthesis.

2. Biomass fuels can be used directly.

Which of the statements given above is/are correct?

(a) 1 only

(b) 2 only

(c) Both 1 and 2

(d) Neither 1 nor 2

Solution: C

Biomass [Conventional Source]

• Biomass is a renewable energy resource derived from plant and animal waste.

• The energy from biomass (biomass conversion) is released on burning or

breaking the chemical bonds of organic molecules formed during photosynthesis.

• Biomass fuels can be used directly or they can be transformed into more

convenient form and then used.

Sources of biomass

• By-products from the timber industry, agricultural crops and their byproducts,

raw material from the forest, major parts of household waste and wood.

• Solid Biomass fuels: Wood logs and wood pellets, charcoal, agricultural waste

(stalks and other plant debris), animal waste (dung), aquatic plants (kelp and

water hyacinths) urban waste (paper, cardboard and other combustible

materials).

62. Consider the following statements regarding Graphite

1. Graphite is a naturally-occurring form of crystalline carbon.

Page 50: INSTA Revision Plan 3.0 - 2020 INSTA Tests - INSIGHTSIAS...2020/08/04  · palm oil. Palm oil contributes 70% of vegetable oil import and is one of the cheapest oil due to high productivity

www.insightsonindia.com 48 INSTA Revision 3.0

INSIGHTSIAS SIMPLYFYING IAS EXAM PREPARATION

2. Graphite is a non-metal and it is the only non-metal that can conduct

electricity.

3. Graphite is hard and cleaves with very high pressure.

Which of the statements given above is/are correct?

(a) 1 and 2 only

(b) 1 and 3 only

(c) 2 and 3 only

(d) 1, 2 and 3

Solution: A

Graphite

• Graphite is a naturally-occurring form of crystalline carbon.

• It is also known as plumbago or black lead.

• The carbon content in Graphite is never less than 95%.

• Graphite may be considered the highest grade of coal, just above anthracite.

Carbon content in Peat < Lignite < Bituminous < Anthracite < Graphite < Diamond

• It is not normally used as fuel because it is difficult to ignite.

• It is found in metamorphic and igneous rocks.

• Graphite is extremely soft, cleaves [splits into layers] with very light pressure.

• It is extremely resistant to heat and is highly unreactive.

• Most of the graphite is formed at convergent plate boundaries where organic-

rich shales and limestones were subjected to metamorphism due to heat and

pressure.

• Metamorphism produces marble, schist and gneiss that contains tiny crystals

and flakes of graphite.

• Some graphite forms from the metamorphism of coal seams. This graphite is

known as “amorphous graphite”.

• Graphite is a non-metal and it is the only non-metal that can conduct

electricity.

63. The ‘Kunming Initiative’, sometimes seen in news is related to:

(a) The South Asian Free Trade Area (SAFTA)

(b) The International North–South Transport Corridor (INSTC)

Page 51: INSTA Revision Plan 3.0 - 2020 INSTA Tests - INSIGHTSIAS...2020/08/04  · palm oil. Palm oil contributes 70% of vegetable oil import and is one of the cheapest oil due to high productivity

www.insightsonindia.com 49 INSTA Revision 3.0

INSIGHTSIAS SIMPLYFYING IAS EXAM PREPARATION

(c) Bangladesh, China, India and Myanmar (BCIM) economic corridor

(d) East Asia Summit (EAS)

Solution: C

The four member countries of the Kunming Initiative that was rechristened as the

BCIM Forum (Bangladesh, India, China, and Myanmar) for regional cooperation is

a Track II initiative that was given Track I coordination in 2011.

The Kunming Initiative was formed keeping in mind the fact that a regional outlook

on the concerns of development, security and prosperity was undoubtedly more

beneficial than striking out one’s own.

https://www.business-standard.com/article/news-ians/china-drops-bcim-from-

bri-projects-list-119042800540_1.html

64. Which of the following are the coalfields in Jharkhand?

1. Korba coalfield

2. Jharia coalfield

3. Rajmahal coalfield

4. Talcher field

Select the correct answer using the code given below:

(a) 1 and 2 only

(b) 2 and 3 only

(c) 2 and 4 only

(d) 1 and 4 only

Solution: B

Gondwana Coalfields in Chhattisgarh

• Korba coalfield

• Birampur coalfield

• Hasdo-Arand coalfield

• Chirmiri coalfield

• Lakhanpur coalfield

• Jhilmili coalfield

• Johilla coalfield

• Sonhat coalfield

• Tatapani-Ramkota coalfields

Page 52: INSTA Revision Plan 3.0 - 2020 INSTA Tests - INSIGHTSIAS...2020/08/04  · palm oil. Palm oil contributes 70% of vegetable oil import and is one of the cheapest oil due to high productivity

www.insightsonindia.com 50 INSTA Revision 3.0

INSIGHTSIAS SIMPLYFYING IAS EXAM PREPARATION

Gondwana Coalfields in Jharkhand

• Jharia coalfield

• Jayanti coalfields

• Bokaro coalfield

• West Bokaro [900 m deep]

• East Bokaro [600 m deep]: Hazaribagh district

• Girdih (Karharbari) coalfield

• Karanpura and Ramgarh coalfields

• Auranga coalfield: Palamu district

• Hutar coalfield

• Deltenganj coalfield

• Devgarh coalfields: Dumka district

• Rajmahal coalfield, Rajmahal hills

Gondwana Coalfields in Odisha

• Talcher field: Talcher town to Rairkhol in Dhenkanal and Sambalpur districts

• Rampur-Himgir coalfield: Sambalpur and Sundargarh

• Ib river coalfield: Sambalpur and Jharsuguda district

Gondwana Coalfields in Madhya Pradesh

• Singrauli (Waidhian) coalfield: Sidhi and Shandol districts

• Pench-Kanhan-Tawa: Chhindwara district

• Sohagpur coalfield: Shandol district

• Umaria coalfield: Umaria district

65. Consider the following statements regarding the Malaria disease:

1. It is caused by Plasmodium parasites.

2. It is preventable and curable disease.

Which of the statements given above is/are correct?

(a) 1 only

(b) 2 only

(c) Both 1 and 2

(d) Neither 1 nor 2

Solution: C

Malaria is caused by Plasmodium parasites. The parasites are spread to people

through the bites of infected female Anopheles mosquitoes, called “malaria

vectors.” There are 5 parasite species that cause malaria in humans, and 2 of these

species – P. falciparum and P. vivax – pose the greatest threat.

Page 53: INSTA Revision Plan 3.0 - 2020 INSTA Tests - INSIGHTSIAS...2020/08/04  · palm oil. Palm oil contributes 70% of vegetable oil import and is one of the cheapest oil due to high productivity

www.insightsonindia.com 51 INSTA Revision 3.0

INSIGHTSIAS SIMPLYFYING IAS EXAM PREPARATION

It is preventable and curable.

https://www.who.int/news-room/fact-sheets/detail/malaria

https://www.businesstoday.in/pti-feed/icmr-launches-mera-india-to-eliminate-

malaria-from-india-by-2030/story/340653.html

66. Which of the following places has/have copper reserves

1. Balaghat district, Madhya Pradesh

2. Khetri-Singhana belt in Rajasthan

3. Singhbhum, Jharkhand

Select the correct answer using the code given below:

(a) 1 and 2 only

(b) 1 and 3 only

(c) 2 and 3 only

(d) 1, 2 and 3

Solution: D

Copper Reserves in India- 46 million tonnes.

• Rajasthan (50%)

• Madhya Pradesh (24%)

• Jharkhand (19%)

The rest 7 per cent in AP, Gujarat, Haryana, Karnataka etc.

Madhya Pradesh

• 1st in production [59.85 %].

• Malanjkhand copper mines of Balaghat district are the most important ones.

• Reserves of moderate size are also found in Betul district.

Rajasthan

• 2nd in production [28%]

• Found along the Aravali range.

• Ajmer, Alwar, Bhilwara, Chittaurgarh, Dungarpur, Jaipur, Jhunjhunu, Pali,

Sikar, Sirohi and Udaipur districts.

Page 54: INSTA Revision Plan 3.0 - 2020 INSTA Tests - INSIGHTSIAS...2020/08/04  · palm oil. Palm oil contributes 70% of vegetable oil import and is one of the cheapest oil due to high productivity

www.insightsonindia.com 52 INSTA Revision 3.0

INSIGHTSIAS SIMPLYFYING IAS EXAM PREPARATION

• Khetri-Singhana belt in Jhunjhunu district is the most important copper

producing area.

Jharkhand

• 3rd in production [11 %].

• Singhbhum is the most important copper producing district.

• Found in Hazaribagh district, Santhal Parganas and Palamu districts.

67. Zemu Glacier is located in the State of:

(a) Sikkim

(b) Arunachal Pradesh

(c) Himachal Pradesh

(d) Uttarakhan

Solution: A

Zemu Glacier is the largest glacier in the Eastern Himalaya. It is about 26

kilometres in length and is located at the base of Kangchenjunga in the Himalayan

region of Sikkim, India. The Zemu Glacier drains the east side of Kanchenjunga,

the world’s third highest mountain.

68. Manjeera Wildlife sanctuary, sometimes seen in news, located in the state of:

(a) Telangana

(b) Andhra Pradesh

(c) Karnataka

(d) Maharashtra

Solution: A

Manjira wildlife sanctuary is a wildlife sanctuary and reservoir located in

Sangareddy district of Telangana State, India. Originally a crocodile sanctuary,

today more than 70 species of birds are spotted here and is home for the vulnerable

species mugger crocodile.

https://timesofindia.indiatimes.com/city/hyderabad/as-manjeera-and-singur-dry-

up-water-board-falls-back-on-citys-twin-reservoirs/articleshow/69604449.cms

Page 55: INSTA Revision Plan 3.0 - 2020 INSTA Tests - INSIGHTSIAS...2020/08/04  · palm oil. Palm oil contributes 70% of vegetable oil import and is one of the cheapest oil due to high productivity

www.insightsonindia.com 53 INSTA Revision 3.0

INSIGHTSIAS SIMPLYFYING IAS EXAM PREPARATION

69. Consider the following statements regarding Polymetallic Nodules

1. Polymetallic nodules are rounded accretions of manganese and iron

hydroxides that cover vast areas of the seafloor

2. India signed a 15-year contract for exploration of Polymetallic Nodules

(PMN) in the Central Indian Ocean Basin with the International Seabed

Authority (ISA)

Which of the statements given above is/are correct?

(a) 1 only

(b) 2 only

(c) Both 1 and 2

(d) Neither 1 nor 2

Solution: C

Polymetallic nodules are rounded accretions of manganese and iron hydroxides

that cover vast areas of the seafloor, but are most abundant on abyssal plains at

water depths of 4000-6500 metres. They form through the aggregation of layers of

iron and manganese hydroxides around a central particle (such as a shell or small

rock fragment), and range in size from a few millimeters to tens of centimeters.

The first deep-sea venture of the National Institute of Oceanography was started

in 1982, for the exploration for nodules and India recognized as a “Pioneer

Investor”. In 1995, India ratifies the UNCLOS III convention and India elected as

the council member of the International Seabed Authority.

India signed a 15-year contract for exploration of Polymetallic Nodules (PMN) in

the Central Indian Ocean Basin with the International Seabed Authority (ISA) on

25th March, 2002

https://www.eu-midas.net/science/nodules

https://www.thehindu.com/sci-tech/science/why-is-india-pulled-to-deep-sea-

mining/article28809029.ece

70. Consider the following statements regarding the Bt Brinjal:

1. It was developed by the Indian Council of Agriculture Research (ICAR).

2. It incorporates the cry1Ac gene expressing insecticidal protein.

Which of the following statements given above is/are correct?

(a) 1 only

(b) 2 only

(c) Both 1 and 2

Page 56: INSTA Revision Plan 3.0 - 2020 INSTA Tests - INSIGHTSIAS...2020/08/04  · palm oil. Palm oil contributes 70% of vegetable oil import and is one of the cheapest oil due to high productivity

www.insightsonindia.com 54 INSTA Revision 3.0

INSIGHTSIAS SIMPLYFYING IAS EXAM PREPARATION

(d) Neither 1 nor 2

Solution: B

Fruit and Shoot Borer (FSB)-resistant brinjal or Bt brinjal was developed using a

transformation process similar to the one used in the development of Bt cotton, a

biotech crop that was planted on 7.6 million hectares in India in 2008. Bt brinjal

incorporates the cry1Ac gene expressing insecticidal protein to confer resistance

against FSB. The cry1Ac gene is sourced from the soil bacterium Bacillus

thuringiensis (Bt).

Bt Brinjal was developed by the Maharashtra Hybrid Seeds Company (Mahyco).

The company used a DNA construct containing the cry1Ac gene, a CaMV 35S

promoter and the selectable marker genes nptII and aad, to transform young

cotyledons of brinjal plants.

https://www.thehindubusinessline.com/economy/agri-business/bt-brinjal-being-

illegally-grown-in-haryana/article26945204.ece

71. Consider the following pairs

Hills State/Region 1. Ajanta Range : Maharashtra 2. Garo Hills : Meghalaya 3. Dafla Hills : Tripura

Which of the pairs given above is/are matched correctly?

(a) 1 and 3 only

(b) 2 only

(c) 1 and 2 only

(d) 3 only

Solution: C

Page 57: INSTA Revision Plan 3.0 - 2020 INSTA Tests - INSIGHTSIAS...2020/08/04  · palm oil. Palm oil contributes 70% of vegetable oil import and is one of the cheapest oil due to high productivity

www.insightsonindia.com 55 INSTA Revision 3.0

INSIGHTSIAS SIMPLYFYING IAS EXAM PREPARATION

72. Consider the following statements

1. India imports coking coal.

2. Anthracite coal is largely found in Jharkhand and Bihar

Which of the statements given above is/are correct?

(a) 1 only

(b) 2 only

(c) Both 1 and 2

(d) Neither 1 nor 2

Solution: A

Page 58: INSTA Revision Plan 3.0 - 2020 INSTA Tests - INSIGHTSIAS...2020/08/04  · palm oil. Palm oil contributes 70% of vegetable oil import and is one of the cheapest oil due to high productivity

www.insightsonindia.com 56 INSTA Revision 3.0

INSIGHTSIAS SIMPLYFYING IAS EXAM PREPARATION

Coal is originated from organic matter wood. When large tracts of forests are buried

under sediments, wood is burnt and decomposed due to heat from below and

pressure from above. The phenomenon makes coal but takes centuries to complete.

As per the present import policy, coal can be freely imported (under Open General

Licence) by the consumers themselves considering their needs based on their

commercial prudence. Coking Coal is being imported by Steel Authority of India

Limited (SAIL) and other Steel manufacturing units mainly to bridge the gap

between the requirement and indigenous availability and to improve the quality of

production.

Anthracite: It is the best quality of coal with highest calorific value and carries

80 to 95% carbon content. It ignites slowly with a blue flame and found in small

quantities in Jammu and Kashmir.

73. Consider the following statements regarding the food additive chemical

Melamine:

1. It is high in Nitrogen.

2. It is added to inflate the apparent protein content of food products.

3. There are no approved direct food uses for melamine.

Which of the statements given above is/are correct?

(a) 1 and 2 only

(b) 2 and 3 only

(c) 1 and 3 only

(d) 1, 2 and 3

Solution: D

Melamine is illegally added to inflate the apparent protein content of food

products. Because it is high in nitrogen, the addition of melamine to a food

artificially increases the apparent protein content as measured with standard tests.

There are no approved direct food uses for melamine, nor are there any

recommendations in the Codex Alimentarius.

https://www.livemint.com/politics/policy/government-extends-ban-on-import-of-

milk-products-from-china-1556036439186.html

https://www.who.int/foodsafety/areas_work/chemical-risks/melamine/en/

74. Consider the following statements

1. The Northeastern Plateau extension of the main peninsular plateau.

Page 59: INSTA Revision Plan 3.0 - 2020 INSTA Tests - INSIGHTSIAS...2020/08/04  · palm oil. Palm oil contributes 70% of vegetable oil import and is one of the cheapest oil due to high productivity

www.insightsonindia.com 57 INSTA Revision 3.0

INSIGHTSIAS SIMPLYFYING IAS EXAM PREPARATION

2. Meghalaya plateau is rich in mineral resources like coal, iron ore,

sillimanite, limestone and uranium.

3. Shiwaliks ranges are composed of unconsolidated sediments.

Which of the statements given above is/are correct?

(a) 1 and 2 only

(b) 2 and 3 only

(c) 1 and 3 only

(d) 1, 2 and 3

Solution: D

The Northeastern Plateau

• In fact it is an extension of the main peninsular plateau. It is believed that due

to the force exerted by the northeastward movement of the Indian plate at the

time of the Himalayan origin, a huge fault was created between the Rajmahal

hills and the Meghalaya plateau.

• Later, this depression got filled up by the deposition activity of the numerous

rivers.

• An extension of this is also seen in the Karbi Anglong hills of Assam. Similar

to the Chotanagpur plateau, the Meghalaya plateau is also rich in mineral

resources like coal, iron ore, sillimanite, limestone and uranium. This area

receives maximum rainfall from the south west monsoon.

• The outer-most range of the Himalayas is called the Shiwaliks. They extend

over a width of 10-50 Km and have an altitude varying between 900 and 1100

metres. These ranges are composed of unconsolidated sediments brought down

by rivers from the main Himalayan ranges located farther north.

75. Consider the following statements regarding the Banking Ombudsman Scheme:

1. The Banking Ombudsman is a senior official appointed by the Reserve Bank

of India.

2. It was introduced under the Banking Regulation Act, 1949.

3. All Scheduled Commercial Banks, Regional Rural Banks and Scheduled

Primary Co-operative Banks are covered under the Scheme.

Which of the statements given above is/are correct?

(a) 1 and 2 only

(b) 2 and 3 only

(c) 1 and 3 only

Page 60: INSTA Revision Plan 3.0 - 2020 INSTA Tests - INSIGHTSIAS...2020/08/04  · palm oil. Palm oil contributes 70% of vegetable oil import and is one of the cheapest oil due to high productivity

www.insightsonindia.com 58 INSTA Revision 3.0

INSIGHTSIAS SIMPLYFYING IAS EXAM PREPARATION

(d) 1, 2 and 3

Solution: D

The Banking Ombudsman Scheme enables an expeditious and inexpensive forum

to bank customers for resolution of complaints relating to certain services rendered

by banks. The Banking Ombudsman Scheme is introduced under Section 35 A of

the Banking Regulation Act, 1949 by RBI with effect from 1995.

The Banking Ombudsman is a senior official appointed by the Reserve Bank of

India to redress customer complaints against deficiency in certain banking services.

All Scheduled Commercial Banks, Regional Rural Banks and Scheduled Primary

Co-operative Banks are covered under the Scheme.

https://www.livemint.com/industry/banking/66-of-complaints-mutually-settled-

banking-ombudsman-report-1556162402257.html

DAY – 16

76. Consider the following statements regarding PVTG’s

1. Ministry of Tribal Affairs categories tribal groups into Particularly

Vulnerable Tribal Groups (PVTG)s.

2. 75 tribal groups have been categorized as Particularly Vulnerable Tribal

Groups (PVTG)s across 18 States and UT of A&N Islands.

Which of the statements given above is/are correct?

(a) 1 only

(b) 2 only

(c) Both 1 and 2

(d) Neither 1 nor 2

Solution: B

75 tribal groups have been categorized by Ministry of Home Affairs as Particularly

Vulnerable Tribal Groups (PVTG)s. PVTGs reside in 18 States and UT of A&N

Islands. The Ministry of Tribal Affairs implements the Scheme of “Development of

Particularly Vulnerable Tribal Groups (PVTGs)” exclusively for them. Under the

scheme, Conservation-cum-Development (CCD)/Annual Plans are to be prepared by

each State/UT for their PVTGs based on their need assessment, which are then

appraised and approved by the Project Appraisal Committee of the Ministry. Activities

for development of PVTGs are undertaken in Sectors of Education, Health, Livelihood

Page 61: INSTA Revision Plan 3.0 - 2020 INSTA Tests - INSIGHTSIAS...2020/08/04  · palm oil. Palm oil contributes 70% of vegetable oil import and is one of the cheapest oil due to high productivity

www.insightsonindia.com 59 INSTA Revision 3.0

INSIGHTSIAS SIMPLYFYING IAS EXAM PREPARATION

and Skill Development, Agricultural Development , Housing & Habitat, Conservation

of Culture etc.

https://tribal.nic.in/pvtg.aspx

77. Which of the following criteria is followed for determination of PVTGs?

1. A pre-agriculture level of technology.

2. A stagnant or declining population.

3. Extremely low literacy.

4. A subsistence level of economy.

Select the correct answer using the code given below:

(a) 1, 2 and 3 only

(b) 1, 2 and 4 only

(c) 2, 3 and 4 only

(d) 1, 2, 3 and 4

Solution: D

About ‘Particularly Vulnerable Tribal Groups (PVTGs)’:

• PVTGs are more vulnerable among the tribal groups.

• They have declining or stagnant population, low level of literacy, pre-agricultural

level of technology and are economically backward.

• They generally inhabit remote localities having poor infrastructure and

administrative support.

Identification:

• In 1975, the Government of India initiated to identify the most vulnerable tribal

groups as a separate category called PVTGs and declared 52 such groups, while

in 1993 an additional 23 groups were added to the category, making it a total of

75 PVTGs out of 705 Scheduled Tribes, spread over 18 states and one Union

Territory (A&N Islands) in the country (2011 census).

• Among the 75 listed PVTG’s the highest number are found in Odisha (13),

followed by Andhra Pradesh (12).

Scheme for development of PVTGs:

• The Ministry of Tribal Affairs implements the Scheme of “Development of

Particularly Vulnerable Tribal Groups (PVTGs)” exclusively for them.

• Under the scheme, Conservation-cum-Development (CCD)/Annual Plans are to

be prepared by each State/UT for their PVTGs based on their need assessment,

Page 62: INSTA Revision Plan 3.0 - 2020 INSTA Tests - INSIGHTSIAS...2020/08/04  · palm oil. Palm oil contributes 70% of vegetable oil import and is one of the cheapest oil due to high productivity

www.insightsonindia.com 60 INSTA Revision 3.0

INSIGHTSIAS SIMPLYFYING IAS EXAM PREPARATION

which are then appraised and approved by the Project Appraisal Committee of

the Ministry.

• Priority is also assigned to PVTGs under the schemes of Special Central

Assistance (SCA) to Tribal Sub-Scheme(TSS), Grants under Article 275(1) of the

Constitution, Grants-in-aid to Voluntary Organisations working for the welfare

of Schedule Tribes and Strengthening of Education among ST Girls in Low

Literacy Districts.

The criteria followed for determination of PVTGs are as under:

1. A pre-agriculture level of technology.

2. A stagnant or declining population.

3. Extremely low literacy.

4. A subsistence level of economy.

78. Consider the following statements regarding the administration of union

territory Puducherry:

1. Legislative Assembly can disapprove the ordinance passed by the

administrator (LG) in Puducherry.

2. Puducherry’s elected government cannot make laws concerning public

order, police and land rights.

Which of the statements given above is/are correct?

(a) 1 only

(b) Gulf of Oman

(c) Both 1 and 2

(d) Neither 1 nor 2

Solution: A

• Puducherry is a union territory which is governed by Article 239A of the

Constitution.

• NCT of Delhi, meanwhile, is governed by Article 239AA of the Constitution

which imposes restrictions on the law-making power of the elected legislature of

Delhi.

• According to Article 239AA, the elected legislature of NCT of Delhi cannot

enact laws relating to entries 1,2 and 18 of the state list.

• It means that Delhi’s elected government cannot make laws concerning public

order, police and land rights. This is not the case with Puducherry.

Page 63: INSTA Revision Plan 3.0 - 2020 INSTA Tests - INSIGHTSIAS...2020/08/04  · palm oil. Palm oil contributes 70% of vegetable oil import and is one of the cheapest oil due to high productivity

www.insightsonindia.com 61 INSTA Revision 3.0

INSIGHTSIAS SIMPLYFYING IAS EXAM PREPARATION

• According to 239A which governs the administration of Puducherry, the

administrator (LG) does not have power to promulgate any ordinance in

Puducherry.

• Even if LG does it, it will not stand if the Legislative Assembly decides to

disapprove it. This emphasizes the supremacy of the legislature over the

administrator in Puducherry, unlike the restrictions on the government of NCT

Delhi.

• According to Article 240, even the rights of the President of India is curtailed

to the point that he/she cannot issue ordinances to suit their whims and

fancies.

79. Consider the following pairs of PVTGs and the states they reside

1. Siddi : Gujarat 2. Jenu Kuruba : Karnataka 3. Kadar : Andhra Pradesh

Which of the pairs given above is/are correctly matched?

(a) 1 and 2 only

(b) 1 and 3 only

(c) 2 and 3 only

(d) 1, 2 and 3

Solution: A

The Siddi are recognized as a scheduled tribe in 3 states and 1 union territory: Goa,

Gujarat, Karnataka and Daman and Diu.

The Jennu Kurumba are spread out mainly along the border forests of Kerala, Tamil

Nadu and Karnataka. They are also referred to by the names ‘Then Kurumba’ or

‘Kattu Naikar.’

Page 64: INSTA Revision Plan 3.0 - 2020 INSTA Tests - INSIGHTSIAS...2020/08/04  · palm oil. Palm oil contributes 70% of vegetable oil import and is one of the cheapest oil due to high productivity

www.insightsonindia.com 62 INSTA Revision 3.0

INSIGHTSIAS SIMPLYFYING IAS EXAM PREPARATION

80. Chitwan National Park, sometimes seen in news, located in:

(a) Bhutan

(b) Bangladesh

(c) Nepal

(d) Myanmar

Solution: C

Chitwan National Park is a preserved area in the Terai Lowlands of south-central

Nepal, known for its biodiversity. Its dense forests and grassy plains are home to

rare mammals like one-horned rhinos and Bengal tigers. The park shelters

numerous bird species, including the giant hornbill. Dugout canoes traverse the

northern Rapti River, home to crocodiles. Inside the park is Balmiki Ashram, a Hindu

pilgrimage site.

https://www.thehindu.com/news/national/no-indo-nepal-pact-on-tigers-

yet/article26122318.ece

81. Which of the following countries border Baltic Sea?

1. Norway

2. Sweden

3. Latvia

4. Poland

Select the correct answer using the code given below:

(a) 1, 2 and 3 only

(b) 1, 2 and 4 only

(c) 2, 3 and 4 only

(d) 1, 2, 3 and 4

Solution: C

Page 65: INSTA Revision Plan 3.0 - 2020 INSTA Tests - INSIGHTSIAS...2020/08/04  · palm oil. Palm oil contributes 70% of vegetable oil import and is one of the cheapest oil due to high productivity

www.insightsonindia.com 63 INSTA Revision 3.0

INSIGHTSIAS SIMPLYFYING IAS EXAM PREPARATION

82. Consider the following statements regarding the Brus tribes:

1. The Brus, are spread across the northeastern states of Tripura, Assam,

Manipur, and Mizoram.

2. In Tripura, they are recognized as a Particularly Vulnerable Tribal Group.

3. In Mizoram, they have been targeted by groups that do not consider them

indigenous to the state.

Which of the statements given above is/are correct?

(a) 1 and 2 only

(b) 1 and 3 only

Page 66: INSTA Revision Plan 3.0 - 2020 INSTA Tests - INSIGHTSIAS...2020/08/04  · palm oil. Palm oil contributes 70% of vegetable oil import and is one of the cheapest oil due to high productivity

www.insightsonindia.com 64 INSTA Revision 3.0

INSIGHTSIAS SIMPLYFYING IAS EXAM PREPARATION

(c) 2 and 3 only

(d) 1, 2 and 3

Solution: D

The centre has signed a historic pact for permanent solution of Bru refugees’

issue.

The agreement is between Union Government, Governments of Tripura and

Mizoram and Bru-Reang representatives to end the 23-year old Bru-Reang refugee

crisis.

Highlights of the agreement:

• Under the agreement, the centre has announced a package of Rs. 600 crores

under this agreement.

• As per the agreement the Bru tribes would be given land to reside in Tripura.

• A fixed deposit of Rs. 4 lakh will be given to each family as an amount of

government aid. They will be able to withdraw this amount after two years.

• Each of the displaced families will be given 40×30 sq ft residential plots.

• Apart from them, each family will be given Rs. 5,000 cash per month for two

years.

• The agreement highlights that each displaced family will also be given free ration

for two years and aid of Rs. 1.5 lakh to build their houses.

Significance of the government:

• This agreement will bring a permanent solution for the rehabilitation of

thousands of Bru-Reang people in Tripura. The government believes that this

agreement will bring a bright future for them. Bru-Reang people will be able to

enjoy the benefits of all social-welfare schemes of governments.

Background:

• More than 30,000 Bru tribes who fled Mizoram, are residing in Tripura’s refugee

camps.

Who are Brus?

• The Brus, also referred to as the Reangs, are spread across the northeastern

states of Tripura, Assam, Manipur, and Mizoram.

• In Tripura, they are recognised as a Particularly Vulnerable Tribal Group. In

Mizoram, they have been targeted by groups that do not consider them

indigenous to the state.

Page 67: INSTA Revision Plan 3.0 - 2020 INSTA Tests - INSIGHTSIAS...2020/08/04  · palm oil. Palm oil contributes 70% of vegetable oil import and is one of the cheapest oil due to high productivity

www.insightsonindia.com 65 INSTA Revision 3.0

INSIGHTSIAS SIMPLYFYING IAS EXAM PREPARATION

What’s the issue?

• A bout of ethnic violence forced thousands of people from the Bru tribe to leave

their homes in Mizoram.

• The displaced Bru people from Mizoram have been living in various camps in

Tripura since 1997. In 1997, the murder of a Mizo forest guard at the Dampa

Tiger Reserve in Mizoram’s Mamit district allegedly by Bru militants led to a

violent backlash against the community, forcing several thousand people to flee

to neighbouring Tripura.

• The Bru militancy was a reactionary movement against Mizo nationalist groups

who had demanded in the mid-1990s that the Brus be left out of the state’s

electoral rolls, contending that the tribe was not indigenous to Mizoram.

83. Which of the following countries is/are members of the Arctic Council:

1. France

2. Canada

3. Russia

4. Germany

Select the correct answer using the code given below:

(a) 1 and 4 only

(b) 2 and 3 only

(c) 1, 2 and 3 only

(d) 2, 3 and 4 only

Solution: B

The Arctic Council is the leading intergovernmental forum promoting cooperation,

coordination and interaction among the Arctic States, Arctic indigenous communities

and other Arctic inhabitants on common Arctic issues, in particular on issues of

sustainable development and environmental protection in the Arctic.

• It was formally established in 1996.

• The Ottawa Declaration defines these states as Members of the Arctic Council:

Canada, Denmark, Finland, Iceland, Norway, Russia, Sweden and the United

States.

• India is an observer member of the council.

https://indianexpress.com/article/explained/india-re-election-observer-arctic-

council-importance-5727126/

Page 68: INSTA Revision Plan 3.0 - 2020 INSTA Tests - INSIGHTSIAS...2020/08/04  · palm oil. Palm oil contributes 70% of vegetable oil import and is one of the cheapest oil due to high productivity

www.insightsonindia.com 66 INSTA Revision 3.0

INSIGHTSIAS SIMPLYFYING IAS EXAM PREPARATION

84. Consider the following statements regarding Eklavya Model Residential Schools

1. Eklavya Model Residential Schools (EMRS) are set up in States/UTs with

grants under Article 275(1) of the Constitution of India.

2. The scheme is being implemented by the Ministry of Tribal Affairs.

3. As per the budget 2018-19, every block with more than 75% ST population

and at least 20,000 tribal persons will have an Eklavya Model Residential

School by 2022.

Which of the statements given above is/are correct?

(a) 1 and 2 only

(b) 1 and 3 only

(c) 2 and 3 only

(d) 1, 2 and 3

Solution: A

Eklavya Model Residential Schools

• In the context of establishing quality residential schools for the promotion of

education, Eklavya Model Residential Schools (EMRSs) for ST students are set

up in States/UTs with provisioning of funds through “Grants under Article 275(1)

of the Constitution”.

• The establishing of EMRSs is based on demand of the concerned States/UTs with

availability of land as an essential attribute.

• As per the budget 2018-19, every block with more than 50% ST population and

at least 20,000 tribal persons will have an Eklavya Model Residential School by

2022.

Background:

What is Eklavya Model Residential School (EMRS)?

• EMRS is a Government of India scheme for model residential school for Indian

tribals (Scheduled Tribes, ST) across India.

• In the context of the trend of establishing quality residential schools for the

promotion of education in all areas and habitations in the country, the Eklavya

Model Residential Schools (EMRS) for ST students take their place among the

Jawahar Navodaya Vidyalayas, the Kasturba Gandhi Balika Vidyalayas and the

Kendriya Vidyalayas. Eklavya Model Residential Schools (EMRS) are set up in

States/UTs with grants under Article 275(1) of the Constitution of India.

• The scheme is being implemented by the Ministry of Tribal Affairs, Government

of India.

Page 69: INSTA Revision Plan 3.0 - 2020 INSTA Tests - INSIGHTSIAS...2020/08/04  · palm oil. Palm oil contributes 70% of vegetable oil import and is one of the cheapest oil due to high productivity

www.insightsonindia.com 67 INSTA Revision 3.0

INSIGHTSIAS SIMPLYFYING IAS EXAM PREPARATION

Objectives of EMRS:

• Comprehensive physical, mental and socially relevant development of all

students enrolled in each and every EMRS. Students will be empowered to be

change agent, beginning in their school, in their homes, in their village and finally

in a larger context.

• Focus differentially on the educational support to be made available to those in

Standards XI and XII, and those in standards VI to X, so that their distinctive

needs can be met,

• Support the annual running expenses in a manner that offers reasonable

remuneration to the staff and upkeep of the facilities.

• Support the construction of infrastructure that provides education, physical,

environmental and cultural needs of student life.

85. Consider the following statements regarding the 12th century social reformer

Basavanna:

1. He is associated with the emergence of Vaishnavism in Karnataka.

2. He was against the idol worship.

Which of the statements given above is/are correct?

(a) 1 only

(b) 2 only

(c) Both 1 and 2

(d) Neither 1 nor 2

Solution: B

The Virashaiva movement was initiated by Basavanna and his companions like

Allama Prabhu and Akkamahadevi. This movement began in Karnataka in the mid-

twelfth century. The Virashaivas argued strongly for the equality of all human

beings and against Brahmanical ideas about caste and the treatment of women.

They were also against all forms of ritual and idol worship.

86. Consider the following statements regarding Sentinelese

1. The Sentinelese are a negrito tribe who live on the North Sentinel Island of

the Andamans.

2. Their numbers are believed to be less than 150 and as low as 40.

3. Genome studies indicate that the Andaman tribes could have been on the

islands even 30,000 years ago.

Page 70: INSTA Revision Plan 3.0 - 2020 INSTA Tests - INSIGHTSIAS...2020/08/04  · palm oil. Palm oil contributes 70% of vegetable oil import and is one of the cheapest oil due to high productivity

www.insightsonindia.com 68 INSTA Revision 3.0

INSIGHTSIAS SIMPLYFYING IAS EXAM PREPARATION

Which of the statements given above is/are correct?

(a) 1 and 2 only

(b) 1 and 3 only

(c) 2 and 3 only

(d) 1, 2 and 3

Solution: D

• The Government has promulgated various laws/regulations from time to time to

ensure that the rights and well-being of the Sentinelese are safeguarded.

Steps taken to ensure the protection of Sentinelese:

• The entire North Sentinel Island along with 5 km coastal sea from high water

mark is notified as tribal reserve.

• The Government respects their way of life style, therefore, has adopted an ‘eyes-

on and hands-off’ practice to protect and safeguard the Sentinelese tribe.

• A protocol of circumnavigation of the North Sentinel Island has been notified.

The ships and aircrafts of Coast Guard and boats of Marine Police make sorties

around North Sentinel to keep surveillance.

They have been protected under:

• A &N Islands (PAT) Regulation 1956.

• Scheduled Castes and the Scheduled Tribes (Prevention of Atrocities) Act,

1989.

• Restrictions under Foreigner (Restricted Area) Orders, 1963.

• Visa Manual Conditions/Passport Act 1920, Indian Forest Act, 1927 and

Wildlife (Protection) Act, 1972.

Who are the Sentinelese?

• The Sentinelese are a negrito tribe who live on the North Sentinel Island of the

Andamans. The inhabitants are connected to the Jarawa on the basis of physical,

as well as linguistic similarities. Their numbers are believed to be less than 150

and as low as 40.

• Based on carbon dating of kitchen middens by the Anthropological Survey of

India, Sentinelese presence was confirmed in the islands to 2,000 years ago.

Genome studies indicate that the Andaman tribes could have been on the islands

even 30,000 years ago.

Why are they said to be vulnerable?

• It is said they have made little to no advancement in the over 60,000 years and

still live very primitive lives, surviving mainly on fish and coconuts.

Page 71: INSTA Revision Plan 3.0 - 2020 INSTA Tests - INSIGHTSIAS...2020/08/04  · palm oil. Palm oil contributes 70% of vegetable oil import and is one of the cheapest oil due to high productivity

www.insightsonindia.com 69 INSTA Revision 3.0

INSIGHTSIAS SIMPLYFYING IAS EXAM PREPARATION

• They are very vulnerable to germs since they have not had contact with the

outside world. Even a common flu virus carried by a visitor could wipe out the

entire tribe.

• Since the 1960s, there have been a handful of efforts to reach out to the tribe

but all have largely failed. They have repeatedly, aggressively made it clear that

they want to be isolated.

Conclusion:

• Currently, there is a one-size-fits-all policy. For instance, the Sentinelese should

be left alone. The rights and the desires of the Sentinelese need to be respected

and nothing is to be achieved by escalating the conflict and tension.

87. Consider the following statements regarding tribes on Lakshadweep islands

1. Lakshadweep is Muslim majority state in India with approximately 96.58 %

of state population following Islam as their religion.

2. Of the total population of Lakshadweep, 94.5 per cent are Scheduled Tribes

(STs).

Which of the statements given above is/are correct?

(a) 1 only

(b) 2 only

(c) Both 1 and 2

(d) Neither 1 nor 2

Solution: C

Lakshadweep is Muslim majority state in India with approximately 96.58 % of state

population following Islam as their religion. Hinduism is second most popular

religion in state of Lakshadweep with approximately 2.77 % following it. In

Lakshadweep state, Christianity is followed by 0.49 %, Jainism by 0.02 %, Buddhism

by 0.02 % and Sikhism by 0.01 %. Around 0.01 % stated ‘Other Religion’,

approximately 0.10 % stated ‘No Particular Religion’.

Literacy rate in Lakshadweep has seen upward trend and is 91.85 percent as per

2011 population census. Of that, male literacy stands at 95.56 percent while female

literacy is at 87.95 percent. In 2001, literacy rate in Lakshadweep stood at 86.66

percent of which male and female were 92.53 percent and 80.47 percent literate

respectively.

Total area of Lakshadweep is 30 sq. km. Density of Lakshadweep is 2,149 per sq km

which is higher than national average 382 per sq km. In 2001, density of

Lakshadweep was 1,895 per sq km, while nation average in 2001 was 324 per sq

km.

Page 72: INSTA Revision Plan 3.0 - 2020 INSTA Tests - INSIGHTSIAS...2020/08/04  · palm oil. Palm oil contributes 70% of vegetable oil import and is one of the cheapest oil due to high productivity

www.insightsonindia.com 70 INSTA Revision 3.0

INSIGHTSIAS SIMPLYFYING IAS EXAM PREPARATION

The total population of Lakshadweep as per the 2001 Census is 60,650. Of this,

57,321 persons (94.5 per cent) are Scheduled Tribes (STs). Among the states/UTs

Lakshadweep has the highest proportion of ST population besides Mizoram.

According to the 2011 Census, Lakshadweep has a population of 64429 persons.

More than 93% of the population who are indigenous, are Muslims and majority of

them belong to the Shafi School of the Sunni Sect. Malayalam is spoken in all the

islands except Minicoy where people speak Mahl which is written in Divehi script

and is spoken in Maldives also. The entire indigenous population has been classified

as Scheduled Tribes because of their economic and social backwardness. According

to the Scheduled Castes and Scheduled Tribes list (modification orders), 1956, the

inhabitants of Lakshadweep who and both of whose parents were born in these

islands are treated as Scheduled Tribes. There are no Scheduled Castes in this Union

Territory.

88. Consider the following statements regarding the Trans Fats:

1. They can be produced by heating liquid vegetable oils in the presence of

hydrogen gas and a catalyst.

2. It raises the bad cholesterol level in the body.

3. It can contribute to insulin resistance.

Which of the statements given above is/are correct?

(a) 1 and 2 only

(b) 2 and 3 only

(c) 1 and 3 only

(d) 1, 2 and 3

Solution: D

Trans fatty acids, more commonly called trans fats, are made by heating liquid

vegetable oils in the presence of hydrogen gas and a catalyst, a process called

hydrogenation.

Partially hydrogenating vegetable oils makes them more stable and less likely to

become rancid. This process also converts the oil into a solid, which makes them

function as margarine or shortening.

Partially hydrogenated oils can withstand repeated heating without breaking down,

making them ideal for frying fast foods.

For these reasons, partially hydrogenated oils became a mainstay in restaurants and

the food industry – for frying, baked goods, and processed snack foods and

margarine.

Partially hydrogenated oil is not the only source of trans fats in our diets. Trans

fats are also naturally found in beef fat and dairy fat in small amounts.

Page 73: INSTA Revision Plan 3.0 - 2020 INSTA Tests - INSIGHTSIAS...2020/08/04  · palm oil. Palm oil contributes 70% of vegetable oil import and is one of the cheapest oil due to high productivity

www.insightsonindia.com 71 INSTA Revision 3.0

INSIGHTSIAS SIMPLYFYING IAS EXAM PREPARATION

Trans fats are the worst type of fat for the heart, blood vessels, and rest of the body

because they:

• Raise bad LDL (Bad cholesterol) and lower good HDL

• Create inflammation,– a reaction related to immunity – which has been

implicated in heart disease, stroke, diabetes, and other chronic conditions

• Contribute to insulin resistance

• Can have harmful health effects even in small amounts – for each additional 2

percent of calories from trans fat consumed daily, the risk of coronary heart

disease increases by 23 percent.

https://www.thehindu.com/sci-tech/health/who-for-eliminating-industrially-

produced-trans-fats-by-2023/article27077068.ece

89. Which of the following countries border Mediterranean Sea

1. Italy

2. Portugal

3. Algeria

4. Tunisia

Select the correct answer using the code given below:

(a) 1 and 2 only

(b) 1, 3 and 4 only

(c) 2, 3 and 4 only

(d) 1, 2, 3 and 4

Solution: B

Page 74: INSTA Revision Plan 3.0 - 2020 INSTA Tests - INSIGHTSIAS...2020/08/04  · palm oil. Palm oil contributes 70% of vegetable oil import and is one of the cheapest oil due to high productivity

www.insightsonindia.com 72 INSTA Revision 3.0

INSIGHTSIAS SIMPLYFYING IAS EXAM PREPARATION

90. Consider the following statements regarding the Bureau of Indian Standards:

1. It is the National Standard Body of India established under the BIS Act

2016.

2. It works under the aegis of the Ministry of Consumer Affairs, Food & Public

Distribution, Government of India.

Which of the statements given above is/are correct?

(a) 1 only

(b) 2 only

(c) Both 1 and 2

(d) Neither 1 nor 2

Solution: C

BIS is the National Standard Body of India established under the BIS Act 2016 for

the harmonious development of the activities of standardization, marking and quality

certification of goods and for matters connected therewith or incidental thereto. BIS

has been providing traceability and tangibility benefits to the national economy in a

number of ways – providing safe reliable quality goods; minimizing health hazards to

consumers; promoting exports and imports substitute; control over proliferation of

varieties etc. through standardization, certification and testing.

It works under the aegis of the Ministry of Consumer Affairs, Food & Public

Distribution, Government of India.

Page 75: INSTA Revision Plan 3.0 - 2020 INSTA Tests - INSIGHTSIAS...2020/08/04  · palm oil. Palm oil contributes 70% of vegetable oil import and is one of the cheapest oil due to high productivity

www.insightsonindia.com 73 INSTA Revision 3.0

INSIGHTSIAS SIMPLYFYING IAS EXAM PREPARATION

Keeping in view, the interest of consumers as well as the industry, BIS is involved in

various activities as given below:

• Standards Formulation

• Product Certification Scheme

• Compulsory Registration Scheme

• Foreign Manufacturers Certification Scheme

• Hall Marking Scheme

• Laboratory Services

• Laboratory Recognition Scheme

• Sale of Indian Standards

• Consumer Affairs Activities

• Promotional Activities

• Training Services, National & International level

• Information Services

91. Consider the following physical features of Africa

1. Atlas Mountains

2. Ethiopian Highlands

3. Congo Basin

Arrange the above given features from West to East using the code given below:

(a) 1-3-2

(b) 3-2-1

(c) 2-3-1

(d) 2-1-3

Solution: A

Page 76: INSTA Revision Plan 3.0 - 2020 INSTA Tests - INSIGHTSIAS...2020/08/04  · palm oil. Palm oil contributes 70% of vegetable oil import and is one of the cheapest oil due to high productivity

www.insightsonindia.com 74 INSTA Revision 3.0

INSIGHTSIAS SIMPLYFYING IAS EXAM PREPARATION

92. Consider the following statements regarding Tangam tribal community:

1. They reside in the state of Nagaland.

2. Their language Tangam belongs to the Dravidian language family.

Page 77: INSTA Revision Plan 3.0 - 2020 INSTA Tests - INSIGHTSIAS...2020/08/04  · palm oil. Palm oil contributes 70% of vegetable oil import and is one of the cheapest oil due to high productivity

www.insightsonindia.com 75 INSTA Revision 3.0

INSIGHTSIAS SIMPLYFYING IAS EXAM PREPARATION

Which of the statements given above is/are correct?

(a) 1 only

(b) 2 only

(c) Both 1 and 2

(d) Neither 1 nor 2

Solution: D

The Tangams are a little-known community within the larger Adi tribe of Arunachal

Pradesh and reside in the hamlet of Kugging in Upper Siang district’s Paindem circle.

As per the UNESCO World Atlas of Endangered Languages (2009), Tangam — an

oral language that belongs to the Tani group, under the greater Tibeto-Burman

language family — is marked ‘critically endangered’.

https://indianexpress.com/article/explained/the-language-of-the-tangams-with-

just-253-speakers-6503165/

93. Consider the following statements regarding the Basel Convention:

1. It is an independent intergovernmental body.

2. The WCO Secretariat runs the WCO’s day-to-day operations under the

leadership of a Secretary General.

Which of the statements given above is/are correct?

(a) 1 only

(b) 2 only

(c) Both 1 and 2

(d) Neither 1 nor 2

Solution: C

The World Customs Organization (WCO), established in 1952 as the Customs Co-

operation Council (CCC) is an independent intergovernmental body whose mission

is to enhance the effectiveness and efficiency of Customs administrations.

Today, the WCO represents 183 Customs administrations across the globe that

collectively process approximately 98% of world trade. As the global centre of

Customs expertise, the WCO is the only international organization with competence

in Customs matters and can rightly call itself the voice of the international Customs

community.

The WCO’s governing body is the Council.

Page 78: INSTA Revision Plan 3.0 - 2020 INSTA Tests - INSIGHTSIAS...2020/08/04  · palm oil. Palm oil contributes 70% of vegetable oil import and is one of the cheapest oil due to high productivity

www.insightsonindia.com 76 INSTA Revision 3.0

INSIGHTSIAS SIMPLYFYING IAS EXAM PREPARATION

The WCO Secretariat, which is based in Brussels, Belgium, consists of more than

125 staffers from around the world and runs the WCO’s day-to-day operations under

the leadership of a Secretary General.

94. Rabari, Bharvad and Charan tribal communities, recently seen in news, reside

in the state of

(a) Karnataka

(b) Gujarat

(c) Andhra Pradesh

(d) Kerala

Solution: B

The Gujarat government announced that a five-member commission will be formed

to identify the members of Rabari, Bharvad and Charan communities, living in

nesses (tiny, oval-shaped hutments made of mud) of Gir, Barda and Alech areas of

the state, who are eligible to get the benefits of Schedule Tribe (ST) status.

The commission, which will be headed by a retired judge of the high court, comprises

two district judges, one retired forest officer and one retired revenue officer.

https://indianexpress.com/article/india/gujarat-panel-to-be-formed-to-decide-on-

tribal-status-of-3-communities-6497045/

95. Consider the following statements regarding the monkeypox disease:

1. It is a viral zoonotic disease that occurs primarily in tropical rainforest areas

of Central and West Africa.

2. Human-to-human transmission of the virus do not occur.

3. There is currently no specific treatment recommended for monkeypox

Which of the statements given above is/are correct?

(a) 1 only

(b) 2 and 3 only

(c) 1 and 3 only

(d) 2 only

Solution: A

Page 79: INSTA Revision Plan 3.0 - 2020 INSTA Tests - INSIGHTSIAS...2020/08/04  · palm oil. Palm oil contributes 70% of vegetable oil import and is one of the cheapest oil due to high productivity

www.insightsonindia.com 77 INSTA Revision 3.0

INSIGHTSIAS SIMPLYFYING IAS EXAM PREPARATION

Monkeypox is caused by monkeypox virus, a member of the Orthopoxvirus genus

in the family Poxviridae.

• Monkeypox is a viral zoonotic disease that occurs primarily in tropical

rainforest areas of Central and West Africa and is occasionally exported to

other regions.

• Monkeypox typically presents clinically with fever, rash and swollen lymph

nodes.

• Monkeypox virus is mostly transmitted to people from wild animals such as

rodents and primates, but human-to-human transmission also occurs.

• Monkeypox virus is transmitted from one person to another by contact with

lesions, body fluids, respiratory droplets and contaminated materials such

as bedding.

• Typically, up to a tenth of persons ill with monkeypox may die, with most deaths

occurring in younger age groups.

• The clinical presentation of monkeypox resembles that of smallpox, a related

orthopoxvirus infection which was declared eradicated worldwide in 1980.

• There is currently no specific treatment recommended for monkeypox.

Vaccination against smallpox with vaccinia vaccine was demonstrated through

several observational studies to be about 85% effective in preventing

monkeypox. Thus, prior childhood smallpox vaccination may result in a milder

disease course.

• However at the present time, the original (first-generation) smallpox vaccines are

no longer available to the general public. A newer vaccinia-based vaccine was

approved for the prevention of smallpox and monkeypox in 2019 and is also not

yet widely available in the public sector.

96. Consider the following statements regarding Katkari Tribes:

1. It is one of the Particularly Vulnerable Tribal Groups.

2. It is a primitive tribe found in Maharashtra and parts of Gujarat,

Which of the statements given above is/are correct?

(a) 1 only

(b) 2 only

(c) Both 1 and 2

(d) Neither 1 nor 2

Solution: C

Page 80: INSTA Revision Plan 3.0 - 2020 INSTA Tests - INSIGHTSIAS...2020/08/04  · palm oil. Palm oil contributes 70% of vegetable oil import and is one of the cheapest oil due to high productivity

www.insightsonindia.com 78 INSTA Revision 3.0

INSIGHTSIAS SIMPLYFYING IAS EXAM PREPARATION

Katkari is one of the 75 Particularly Vulnerable Tribal Groups, as per the

classification by Ministry of Home Affairs. It is a primitive tribe found in Maharashtra

(Pune, Raigad and Thane districts) and parts of Gujarat. Katkaris are also known

as Kathodis because of their old occupation of making Katha (Catechu) the

thickened sap from wood of Khair ( Acacia catechu).

They serve as agricultural labourers and sell firewood and some jungle fruits. They

also take up fishing for domestic consumption, coal making and brick

manufacturing. Poor literacy rates, health conditions and minimal livelihood

opportunities are some of the issues faced by the tribal people.

https://vikaspedia.in/social-welfare/scheduled-tribes-welfare/the-story-of-

shahapurs-katkari-tribe

97. Nuakhai is an agricultural festival mainly observed in the state of:

(a) Karnataka

(b) Odisha

(c) Andhra Pradesh

(d) Jharkhand

Solution: B

Nuakhai or Navakhai is an agricultural festival mainly observed by people of

Western Odisha and Southern Chhattisgarh in India. Nuakhai is observed to

welcome the new rice of the season.

https://www.hindustantimes.com/india-news/pm-modi-extends-greeting-to-

farmers-on-nuakhai-juhar/story-4l9IcrPrkOJkhpfCMxEaML.html

98. Consider the following statements regarding Asiatic Lion species:

1. Its current range is restricted to the Gir National Park and environs in the

Indian state of Gujarat.

2. It is listed in Schedule I of Wildlife (Protection) Act 1972.

3. It is listed as endangered on IUCN Red List.

Which of the statements given above is/are correct?

(a) 1 and 2 only

(b) 2 and 3 only

(c) 1 and 3 only

(d) 1, 2 and 3

Page 81: INSTA Revision Plan 3.0 - 2020 INSTA Tests - INSIGHTSIAS...2020/08/04  · palm oil. Palm oil contributes 70% of vegetable oil import and is one of the cheapest oil due to high productivity

www.insightsonindia.com 79 INSTA Revision 3.0

INSIGHTSIAS SIMPLYFYING IAS EXAM PREPARATION

Solution: D

Asiatic lions were once distributed upto the state of West Bengal in east and Rewa

in Madhya Pradesh, in central India. At present Gir National Park and Wildlife

Sanctuary is the only abode of the Asiatic lion. The last surviving population of the

Asiatic lions is a compact tract of dry deciduous forest and open grassy scrublands

in southwestern part of Saurashtra region of Gujarat.

Conservation Status:

• Listed in Schedule I of Wildlife (Protection) Act 1972

• Appendix I of CITES

• Endangered on IUCN Red List

https://www.thehindu.com/sci-tech/science/ccmb-scientists-sequence-asiatic-

lion-genome/article27103990.ece

https://www.wwfindia.org/about_wwf/priority_species/threatened_species/asiatic

_lion/

99. Consider the following pairs of festivals and tribes associated with:

Festival/Fairs Tribe 1. Thisam festival : Nagas 2. Baneshwar Fair : Bhils 3. Myoko Festival : Apatanis

Which of the pairs given above is/are correctly matched?

(a) 1 and 2 only

(b) 2 and 3 only

(c) 1 and 3 only

(d) 1, 2 and 3

Solution: D

Thisam Phanit is a cultural practice of celebrating the festival of the dead. It is

one of the major festivals of the Tangkhuls and the Naga Tribes of Manipur, India.

It is observed as an emotional farewell festival to the spirit of the dead, for 12 days

at the end of January every year.

Baneshwar fair is an annual tribal fair held in Dungarpur district in Rajasthan state

of India. The fair is held in the month of January or February at Baneshwar, near

the confluence of the Som and Mahi rivers. This fair is a major fair in tribal

culture and has been described as “the Kumbh mela for the tribals”. It is

predominantly a tribal fair with more than half of the congregation consisting of

Bhils. They revere Baneshwar Mahadev as well as Mavji.

Page 82: INSTA Revision Plan 3.0 - 2020 INSTA Tests - INSIGHTSIAS...2020/08/04  · palm oil. Palm oil contributes 70% of vegetable oil import and is one of the cheapest oil due to high productivity

www.insightsonindia.com 80 INSTA Revision 3.0

INSIGHTSIAS SIMPLYFYING IAS EXAM PREPARATION

Myoko is a celebration of friendship and harmony between various Apatani

villages. This is also the time when new paddy is sown in the terraced fields and

the Apatani pray for a good crop. The festival rotates between the eight Apatani

villages in the state of Arunachal Pradesh.

100. Consider the following statements regarding the Index of Eight Core Industries:

1. It is compiled and released by the Central Statistics Office (CSO)

2. The base year for the Index currently is 2004-05.

Which of the statements given above is/are correct?

(a) 1 only

(b) 2 only

(c) Both 1 and 2

(d) Neither 1 nor 2

Solution: D

The monthly Index of Eight Core Industries (ICI) is a production volume index. ICI

measures collective and individual performance of production in selected eight core

industries viz. Coal, Crude Oil, Natural Gas, Refinery Products, Fertilizers, Steel,

Cement and Electricity.

It is compiled and released by Office of the Economic Adviser (OEA), Department

of Industrial Policy & Promotion (DIPP), and Ministry of Commerce & Industry.

The inter-se weights of these eight industries are largely in alignment with the

respective weight of these industries in the Index of Industrial Production (IIP).

The base year of the ICI has been revised to 2011-12 from 2004-05 in alignment with

the new series of IIP. Combined weight of these eight core industries is 40.27 percent

of IIP with base 2011-12.

Page 83: INSTA Revision Plan 3.0 - 2020 INSTA Tests - INSIGHTSIAS...2020/08/04  · palm oil. Palm oil contributes 70% of vegetable oil import and is one of the cheapest oil due to high productivity